Quiz Neuroscience Part 4 of 4

March 21, 2018 | Author: MedShare | Category: Inhibitory Postsynaptic Potential, Neurotransmitter, Opioid, Benzodiazepine


Comments



Description

Neuroscience – Part 414Mar2009 Neuro #1 – Pharmacology: CNS Drugs (Chapter 21) 1) When enough excitatory postsynaptic potentials (EPSPs) are generated, ____ of the membrane occurs and an action potential is generated. Inhibitory postsynaptic potentials (IPSPs) involve ____ of the membrane and a selective increase in membrane permeability to ____ ions. a) Depolarization; Hyperpolarization; Na+ & K+ b) Depolarization; Hyperpolarization; Clc) Hyperpolarization; Depolarization; Na+ & K+ d) Hyperpolarization; Depolarization; Cl2) Which of the following correctly describes the order in which metabotropic receptors cause the activation of ion channels? a) Receptor => Diffusible messenger => Activating enzyme => G-protein b) Receptor => G-protein => Diffusible messenger => Activating enzyme c) Receptor => G-protein => Activating enzyme => Diffusible messenger d) Receptor => Activating enzyme => Diffusible messenger => G-protein e) Receptor => Activating enzyme => G-protein => Diffusible messenger Match the channel blocker with the ion channel site of action: 3.1) Nicotinic ACh receptor ligand a) Tetrodotoxin (Puffer fish) 3.2) Ca++ voltage N-type channel b) Apamin (Honeybee) 3.3) Na+ voltage channel from outside c) Omega conotoxin (Pacific cone snail) 3.4) Glycine receptor ligand d) !-bungarotoxin (Marine snake) 3.5) Small Ca++ activated K+ channel e) Strychnine (Indian plant) 3.6) Which of the following describes how strychnine affects ion channels? a) Competitive antagonist b) Non-competitive antagonist c) Irreversible antagonist d) Competitive agonist e) Non-competitive agonist 3.7) Which of the following describes how !-bungarotoxin affects ion channels? a) Competitive antagonist b) Non-competitive antagonist c) Irreversible antagonist d) Competitive agonist e) Non-competitive agonist 4) If the resting potential of a given membrane is -70mV, which of the following best describes how an inhibitory postsynaptic potential (IPSP) of 20mV would affect the membrane? a) Depolarize the membrane, moving it to -50mV b) Depolarize the membrane, moving it to -90mV c) Hyperpolarize the membrane, moving it to -50mV d) Hyperpolarize the membrane, moving it to -90mV 5) After exposure to a sarin (GB) nerve agent at a mass-casualty incident scene, a paramedic self-administers a Mark-1 kit, containing auto-injectors of atropine and pralidoxime chloride (2-PAM). The affects seen by exposure to sarin are similar to pyridostigmine, neostigmine, and physostigmine. 2-PAM works as an antidote to the affects of sarin in the neuromuscular junction, effectively: DO NOT DISTRIBUTE -1- Neuroscience – Part 4 14Mar2009 a) Prevents the movement of Ca++ b) Blocks acetylcholine c) Blocks cholinesterase d) Reactivates acetylcholine e) Reactivates cholinesterase 6) Which of the following is a likely and beneficial role of recurrent neurons? a) Cause excessive neurotransmitter release b) Prevent any neurotransmitter release c) Prevent excessive neurotransmitter release d) Prevent too little neurotransmitter release 7.1) Which of the following criteria for neurotransmitter identification would use selective pharmacologic antagonism to determine the manner in which a synaptically released transmitter is acting? a) Localization of neurotransmitter b) Release of neurotransmitter c) Synaptic mimicry of neurotransmitter 7.2) Which of the following criteria for neurotransmitter identification would involve electrically or chemically stimulating a slice of brain tissue and measuring substances near the stimulated area? a) Localization of neurotransmitter b) Release of neurotransmitter c) Synaptic mimicry of neurotransmitter Match the following agonist/receptor type with the transmitter: 8.1) NMDA a) Acetylcholine 8.2) Bromocriptine b) Dopamine 8.3) Nicotine c) GABA 8.4) Bethanechol d) Glutamate 8.5) Kainic acid e) Serotonin 8.6) Taurine, B-alanine f) Glycine 8.7) 2-methyl-5-HT 8.8) Muscimol, Baclofen Neuro #2 – Pharmacology: Sedatives- Hypnotics (Chapter 22) 1) Generalized CNS depression produced by sedative-hypnotics can depress respiratory and vasomotor centers in what part of the brain, leading to coma and death? a) Medulla b) Pons c) Midbrain d) Diencephalon e) Cerebellum 2) Which of the following has an active metabolite seen during degradation? a) Zolpidem b) Zaleplon c) Phenobarbital d) Diazepam e) Flumazenil DO NOT DISTRIBUTE -2- Neuroscience – Part 4 14Mar2009 3.1) Which of the following GABA-A subunits is involved in the benzodiazepine binding site? a) ! subunit b) " subunit c) # subunit d) $ subunit e) % subunit 3.2) The chloride channels is triggered by binding of GABA to which of the following subunits? a) ! & " b) " & # c) # & $ d) $ & % e) % & ! 4.1) Which of the following describes the action of benzodiazepines? a) Increase frequency of GABA-mediated Cl- channel opening b) Increase duration of GABA-mediated Cl- channel opening c) Increase frequency of GABA-mediated Cl- channel opening and block glutamic acid d) Increase duration of GABA-mediated Cl- channel opening and block glutamic acid 4.2) Which of the following describes the action of barbiturates? a) Increase frequency of GABA-mediated Cl- channel opening b) Increase duration of GABA-mediated Cl- channel opening c) Increase frequency of GABA-mediated Cl- channel opening and block glutamic acid d) Increase duration of GABA-mediated Cl- channel opening and block glutamic acid 5) Low dose benzodiazepines and barbiturates exert a calming affect with anxiety relief (sedation). Higher doses will induce sleep (hypnosis). During hypnosis induced by sedative-hypnotics, which of the following is increased? a) Time to fall asleep b) Duration of stage 2 NREM sleep c) Duration of REM sleep d) Duration of stage 4 NREM slow-wave sleep e) Psychomotor and cognitive functions 6.1) Which of the following is NOT characteristic of physiological dependence to a drug when the user stops taking the drug? a) Anxiety b) Tremors c) Compulsion d) Seizures e) Hyper-reflexia 6.2) Sedative-hypnotics are NOT commonly associated with development of tolerance. a) True b) False DO NOT DISTRIBUTE -3- Neuroscience – Part 4 14Mar2009 7.1) Which of the following drugs should be given if the goal is to decreases the latency of sleep onset (fall asleep quickly) without affecting REM sleep? a) Zolpidem b) Zaleplon c) Eszopiclone, low dose d) Eszopiclone, high dose 7.2) Which of the following drugs should be given if the goal is to increase total sleep time without decreasing REM sleep? a) Zolpidem b) Zaleplon c) Eszopiclone, low dose d) Eszopiclone, high dose 8) Flumazenil (Romazicon, Anexate) acts as a competitive antagonist to reverse the CNS depressant effects of which of the following? a) Cocaine b) Ethanol c) Opioids d) Barbiturates e) Benzodiazepines 9.1) Which of the following acts as a partial agonist at brain 5-HT1A receptors, exerting an anxiolytic effect? a) Zolpidem (Ambien) b) Zaleplon (Sonata) c) Eszopiclone (Lunesta) d) Buspirone (BuSpar) 9.2) Which of the following is a non-benzodiazepine used for short-term treatment of insomnia? a) Zolpidem (Ambien) b) Zaleplon (Sonata) c) Eszopiclone (Lunesta) d) Buspirone (BuSpar) 10) Which of the following is NOT true regarding the use of benzodiazepines for treatment of anxiety? a) Relatively high therapeutic index b) Availability of drug(s) to treat overdose c) Low risk of drug interactions based on liver enzyme induction d) Minimal effects on cardiovascular or autonomic functions e) Fast elimination rates 11) Which of the following drugs would have the highest risk of adverse effects (anxiety, orthostatic hypotension, hyper-reflexia, seizures) if it were discontinued abruptly? a) Eszopiclone, 6 hour half-life b) Zolpidem, 1.5-3.5 hour half-life c) Zaleplon, 1-2 hour half-life d) Triazolam, 2-3 hour half-life e) Flurazepam, 40-100 hour half-life DO NOT DISTRIBUTE -4- Neuroscience – Part 4 14Mar2009 12) Which of the following dosages should be used when giving benzodiazepines to elderly patients? a) 3 times younger-adult dose b) 2 times younger-adult dose c) Normal dosage d) 1/2 younger-adult dose e) 1/3 younger-adult dose 13.1) Which of the following drugs has the highest risk of causing impaired cognitive function, impaired psychomotor skills, and excessive daytime sedation in an elderly patient, which could lead to falls and fractures? a) Eszopiclone, 6 hour half-life b) Zolpidem, 1.5-3.5 hour half-life c) Zaleplon, 1-2 hour half-life d) Triazolam, 2-3 hour half-life e) Flurazepam, 40-100 hour half-life 13.2) A college student presents to the Emergency Room with anterograde amnesia stating she believes she was recently given a date-rape drug. Which of the following would most likely show up in blood work if benzodiazepines were suspected? a) Triazolam, low dose b) Triazolam, high dose c) Flunitrazepam, low dose d) Flunitrazepam, high dose e) Eszopiclone 13.3) Extensive clinical use of this of the following can lead to serious CNS effects including behavioral disinhibition, delirium, aggression, and violence? a) Alprazolam b) Diazepam c) Flurazepam d) Midazolam e) Triazolam Neuro #3 – Pharmacology: Alcohols (Chapter 23) 1.1) Which of the following is NOT equivalent to the amount of alcohol that can be metabolized by a typical adult in an hour? a) 7-10g of alcohol b) 3.5oz of wine c) 16oz of beer d) 1oz of distilled 80 proof spirits 1.2) Which of the following is true regarding alcohol dehydrogenase (ADH) metabolism? a) Liver metabolism is lower in women than in men b) Liver metabolism is lower in men than in women c) Brain metabolism is lower in women than in men d) Brain metabolism is lower in men than in women e) Gastrointestinal metabolism is lower in women than in men 1.3) When is the microsomal enzyme-oxidation system (MEOS) activated? a) Initially with consumption of any ethanol, prior to ADH metabolism DO NOT DISTRIBUTE -5- Neuroscience – Part 4 14Mar2009 b) Initially with conjunction with the ADH metabolism c) Initially until the ADH pathway becomes saturated d) After the ADH pathway becomes saturated e) As the concentration of ethanol decreases below 100mg/dL in blood 2.1) Which of the following levels of ethanol in the blood is normally associated with gross drunkenness without loss of consciousness, anesthesia, or coma? a) 20mg/dL b) 70mg/dL c) 150mg/dL d) 300mg/dL e) 500mg/dL 2.2) Which of the following is NOT true regarding the mechanism of action for ethanol or its metabolite acetaldehyde? a) Facilitates action of GABA at GABA-A receptors b) Inhibits the ability of glutamate to activate NMDA receptors c) Modifies the activities of adenylyl cyclase, phospholipase C, and ion channels d) Depression of cardiac contractility e) Vasoconstriction of smooth muscle 3) Which of the following is the most common complication of alcohol abuse? a) Hypoglycemia, gynecomastia, testicular atrophy b) Thiamine deficiency, Wernicke-Korsakoff syndrome c) Pancreatitis d) Hepatic disease e) Cardiovascular disease 4) Which of the following would NOT be used in treatment for acute alcohol intoxication? a) Thiamine administration b) Administering IV dextrose c) Monitoring the airway for aspiration d) Maintaining vital signs e) Placing the patient supine on a backboard to prevent spinal injury Match the drug used to treat alcoholism with the mechanism of action: 5.1) NMDA-R antagonist, GABA-A-R activator a) Disulfiram (Antabuse) 5.2) Inhibits aldehyde dehydrogenase b) Acamprosate (Campral) 5.3) Opioid receptor antagonist c) Naltrexone (Revia, Depade) 6.1) A young boy presents to a rural Emergency Room with bradycardia, shortness of breath, and intermittent seizures. The boy is having trouble seeing and says it looks like he’s “in a snow storm.” The attending physician suspects methanol poisoning. Which of the following should be given to the patient if the goal is to compete with alcohol dehydrogenase and thus suppress metabolism of the methanol? a) IV Fomepizole b) IV Disulfiram c) IV Naloxone d) IV Flumazanil e) IV Ethanol DO NOT DISTRIBUTE -6- Neuroscience – Part 4 14Mar2009 6.2) An infant presents to the emergency room with metabolic acidosis and severely compromised vital signs. A frantic mother believes the child drank antifreeze (ethylene glycol) in the garage. Which of the following should be given to the patient if the goal is to inhibit alcohol dehydrogenase and thus prevent metabolism of ethylene glycol? a) IV Fomepizole b) IV Disulfiram c) IV Naloxone d) IV Flumazanil e) IV Ethanol Neuro #4 – Pharmacology: Anti Seizure Drugs (Chapter 24) 1.1) Which of the following is a tricyclic antidepressant that is considered a drug of choice for partial seizures? a) Phenytoin (Dilantin) b) Carbamazepine (Tegretol) c) Phenobarbital (Luminal) d) Primidone (Mysoline) 1.2) Which of the following is very effective against absence seizures and is associated with both thrombocytopenia and hepatotoxicity? a) Lamotrigine (Lamictal) b) Gabapentin (Neurontin) c) Ethosuximide (Zarotin) d) Valproic acid/Sodium valproate (Depekene) 1.3) Which of the following is a long-acting benzodiazepine with efficacy against absence seizures, may have pronounced sedative effects, and has tolerance issues? a) Clonazepam (Klonopin) b) Lorazepam (Ativan) c) Diazepam (Valium) d) Midazolam (Versed) 1.4) Which of the following is used to treat trigeminal neuralgia (“tic doulouereux”) and is associated with aplastic anemia in some cases? a) Phenytoin (Dilantin) b) Carbamazepine (Tegretol) c) Phenobarbital (Luminal) d) Primidone (Mysoline) 1.5) Which of the following is a drug of choice in treating absence seizures as it lacks the idiosyncratic hepatotoxicity of other such drugs and also has a well established role as the first choice in anti-absence seizures? a) Lamotrigine (Lamictal) b) Gabapentin (Neurontin) c) Ethosuximide (Zarotin) d) Valproic acid/Sodium valproate (Depekene) 1.6) Which of the following is NOT true of phenytoin (Dilantin)? a) Blocks Na+ channels and inhibits repetitive action potentials b) Is effective against partial seizures c) Should be given via the IM route DO NOT DISTRIBUTE -7- Neuroscience – Part 4 14Mar2009 d) Is highly protein bound e) Can cause nystagmus, diplopia, and ataxia 1.7) Which of the following benzodiazepines is a powerful anxiolytic and is used as an adjunctive antiemetic in chemotherapy? a) Clonazepam (Klonopin) b) Lorazepam (Ativan) c) Diazepam (Valium) d) Midazolam (Versed) 1.8) Valproate will cause a two-fold increase in which of the following drug’s half-life? a) Lamotrigine (Lamictal) b) Gabapentin (Neurontin) c) Ethosuximide (Zarotin) 1.9) Which of the following benzodiazepines is NOT used for short surgical procedures such as dental extraction or pre-surgery anti-anxiety? a) Clonazepam (Klonopin) b) Midazolam (Versed) c) Diazepam (Valium) 1.10) Which of the following is an amino acid analog of GABA effective against partial seizures? a) Lamotrigine (Lamictal) b) Gabapentin (Neurontin) c) Ethosuximide (Zarotin) d) Valproic acid/Sodium valproate (Depekene) 1.11) Which of the following is the most commonly used barbiturate, acts on the GABA receptor, and is a common drug of abuse? a) Phenytoin (Dilantin) b) Carbamazepine (Tegretol) c) Phenobarbital (Luminal) d) Primidone (Mysoline) 1.12) What is a commonly seen adverse effect of ethosuximide (Zarotin)? a) Hepatotoxicity b) Aplastic anemia c) Thromboxytopenia d) Gastric distress e) Tachypnea 1.13) Which of the following is NOT true of vigabatrin? a) Is useful in partial seizures b) Is useful in West Syndrome c) Irreversibly inhibits GABA aminotransferase d) Decreases the amount of GABA released at synaptic sites 2.1) Patients who suffer from what type of seizures often wear helmets to prevent serious head injuries? a) Complex partial seizures b) Generalized tonic-clonic (grand mal) seizures c) Absence (petit mal) seizures d) Myoclonic jerking seizures DO NOT DISTRIBUTE -8- Neuroscience – Part 4 14Mar2009 e) Atonic seizures f) None of the above 2.2) What type of seizure always involves the limbic system, involves memory loss of the event, and usually involves automatisms (e.g. lip smacking, fumbling, walking about)? a) Complex partial seizures b) Generalized tonic-clonic (grand mal) seizures c) Absence (petit mal) seizures d) Myoclonic jerking seizures e) Atonic seizures f) None of the above 2.3) In the Emergency Room, a mother rushes to the reception desk distressed that her child is “shaking all over the bed.” Entering the child’s room, the patient is convulsing and has urinary incontinence. Which of the following drugs would NOT be an option for this patient? a) Valproate b) Phenytoin c) Barbiturates d) Carbamazepine e) Ethosuximide 2.4) A mentally retarded infant presents with brief, recurrent myoclonic jerks of the body with sudden flexion and extension of the limbs. Which of the following drugs should be administered via the IM route? a) Clonazepam b) Phenytoin c) Barbiturates d) Carbamazepine e) Corticotrophin 2.5) Which of the following is a drug of choice for atonic seizures? a) Clonazepam b) Phenytoin c) Valproate d) Carbamazepine e) Lamotrigine 2.6) Which of the following would NOT be given for absence status seizures? a) Diazepam b) Lorazepam c) Phenytoin d) Clonazepam e) Midazolam 3) Which of the following teratogenic drugs has been implicated in spina bifida and is not associated with fetal hydantoin syndrome (embryopathy)? a) Phenytoin b) Valproate c) Carbamazepine d) Phenobarbital DO NOT DISTRIBUTE -9- Neuroscience – Part 4 14Mar2009 Neuro #5 – Pharmacology: General Anesthetics (Chapter 25) 1.1) Which of the following inhaled anesthetics is NOT a volatile liquid at room temperature? a) Nitrous oxide b) Halothane c) Enflurane d) Isoflurane e) Desflurane f) Sevoflurane g) Methoxyflurane Match the common intravenous anesthetic with description: 1.2) Used for Rapid Sequence Intubation (RSI) and conscious sedation a) Thiopental 1.3) Opioid analgesic &81x more potent than morphine b) Midazolam 1.4) Very common anesthesia induction agent, opaque white in color c) Fentanyl 1.5) Ultra-short-acting barbiturate sometimes used in interrogation d) Propofol 1.6) Short-acting benzodiazepine derivative with reversal drug available e) Etomidate 2) A patient is being induced by an anesthesiologist prior to surgery and has lost the eyelash reflex, responsiveness to a small electric shock on the face, and has established a regular respiratory pattern. What state of anesthesia is this patient in? a) Stage 1 – Analgesia b) Stage 2 – Excitement c) Stage 3 – Surgical anesthesia d) Stage 4 – Medullary depression 3) Which of the following has the more rapid onset and recovery (elimination via lungs)? a) Nitrous oxide, blood:gas partition coefficient of 0.47 b) Sevoflurane, blood:gas partition coefficient of 0.69 c) Isoflurane, blood:gas partition coefficient of 1.40 d) Halothane, blood:gas partition coefficient of 2.30 e) Methoxyflurane, blood:gas partition coefficient of 12 4.1) Given the blood:gas partition coefficient, which of the following most likely has the highest minimum alveolar concentration (MAC), and thus is the least potent? a) Nitrous oxide, blood:gas partition coefficient of 0.47 b) Sevoflurane, blood:gas partition coefficient of 0.69 c) Isoflurane, blood:gas partition coefficient of 1.40 d) Halothane, blood:gas partition coefficient of 2.30 e) Methoxyflurane, blood:gas partition coefficient of 12 4.2) Which of the following anesthetics, unlike the mechanisms of most other anesthetics, does NOT affect GABA? a) Thiopental b) Midazolam c) Ketamine d) Propofol e) Etomidate 4.3) Which of the following corresponds to a stage III response to anesthetics? a) Interruption of spinothalamic (ALS) tract via neurons in the substantia gelatinosa of the dorsal horn of the spinal cord DO NOT DISTRIBUTE - 10 - Neuroscience – Part 4 14Mar2009 b) Blockage of many small inhibitory neurons in the brain c) Depression of ascending pathways in the reticular activating system d) Depression of respiratory and vasomotor centers of the medulla 4.4) Which of the following is NOT true regarding minimum alveolar concentration (MAC) of general anesthetics? a) At 760mmHg of nitrous oxide, most (>50%) patients are still mobile b) Individual patients required between 0.5 and 1.5 MAC for surgery c) 100% of patients with 1.0 MAC will not respond to a surgical incision d) 0.4 MAC of an IV anesthetic and 0.7 MAC of an inhaled anesthetic will yield an overall 1.1 MAC 4.5) Which of the following anesthetics decreases mean arterial pressure and is arrhythmogenic? a) Halothane b) Enflurane c) Sevoflurane d) Isoflurane 5) Which of the following drugs, like nitrous oxide, dissolves the least in blood? a) Methoxyflurane b) Halothane c) Isoflurane d) Sevoflurane e) Desflurane 6.1) Which of the following drugs is associated with post-op hepatitis (hepatotoxicity)? a) Nitrous oxide b) Halothane c) Isoflurane d) Sevoflurane e) Desflurane 6.2) Which of the following toxic effects of inhaled anesthetics is treated with the Ca++ release blocker dantrolene? a) Hepatotoxicity b) Nephrotoxicity c) Malignant hyperthermia d) Chronic toxicity (mutagenicity) 6.3) Which of the following drugs is associated with nephrotoxicity? a) Methoxyflurane b) Halothane c) Isoflurane d) Sevoflurane e) Desflurane 7) Which of the following works by blocking the NMDA glutamic acid receptor, shows cardiovascular stimulating effects (raises HR, BP, CO), and can produce dissociative anesthesia with perceptual illusions? a) Thiopental b) Midazolam c) Ketamine DO NOT DISTRIBUTE - 11 - Neuroscience – Part 4 14Mar2009 d) Propofol e) Etomidate 8) If rapid recovery is associated with fewer post-operative adverse effects, which of the following inhaled anesthetics would be a poor choice in balance anesthesia (inhaled + IV anesthetic)? a) Nitrous oxide b) Halothane c) Sevoflurane d) Desflurane e) Methoxyflurane 9.1) Which of the following drugs is preferred in patients with cerebral swelling as it does not increase intracranial pressure? a) Etomidate b) Propofol c) Midazolam d) Thiopental e) Fentanyl 9.2) Which of the following is associated with a high incidence of anterograde amnesia (clinically useful effect)? a) Etomidate b) Propofol c) Midazolam d) Thiopental e) Fentanyl 9.3) What drug in combination with N2O and droperidol gives neuroleptanesthesia, which is amnesia and analgesia? a) Etomidate b) Propofol c) Midazolam d) Thiopental e) Fentanyl 9.4) Which of the following is NOT true regarding propofol (Diprivan)? a) Has antiemetic actions b) Is popular as an induction agent c) Is used for anesthesia maintenance d) Causes a decrease in blood pressure e) Is relatively cheap 9.5) Naloxone is a receptor antagonist that can reverse the effects of which of the following? a) Etomidate b) Propofol c) Midazolam d) Thiopental e) Fentanyl 9.6) Which of the following drugs is used to induce patients with limited cardiovascular reserve and can cause adrenocortical suppression? DO NOT DISTRIBUTE - 12 - Neuroscience – Part 4 a) Etomidate b) Propofol c) Midazolam d) Thiopental e) Fentanyl 10) Which of the following is NOT true regarding conscious sedation? a) Alleviates anxiety b) Alleviates pain c) Causes motor paralysis d) Allows for maintenance of patent airway e) Allows for the ability to respond to verbal commands 14Mar2009 Neuro #6 – Pharmacology: Local Anesthetics (Chapter 26) 1) Which of the following is the most potent amide (relative to procaine) as well as longest acting? a) Cocaine b) Tetracaine c) Lidocaine d) Prilocaine e) Bupivicaine 2.1) Most local anesthetics are weak ____, with the neutral form allowing for penetration of nerve membranes. At physiologic pH, the ____ form of the anesthetic prevails. a) Acids; Anionic (-) b) Acids; Cationic (+) c) Bases; Anionic (-) d) Bases; Cationic (+) 2.2) If a local anesthetic is injected into infected tissue, which has a low extracellular pH, which of the following is true? a) The anesthetic is more effective b) The anesthetic is equally effective c) The anesthetic is less effective 3) Which of the following types of nerve fibers is the least susceptible to local anesthetics, which block voltage gated sodium channels? a) Delta pain/temperature fibers b) Dorsal root pain fibers c) Type B preganglionic autonomic fibers d) Alpha proprioceptive/motor fibers e) Sympathetic postganglionic fibers 4) Hypercalcemia would partially ____ the action of local anesthetics and hyperkalemia would partially ____ the action of local anesthetics. a) Agonize; Agonize b) Agonize; Antagonize c) Antagonize; Antagonize d) Antagonize; Agonize 5) Which of the following cases is the most susceptible to local anesthetics? a) Smaller fibers, heavily myelinated, located circumferentially DO NOT DISTRIBUTE - 13 - Neuroscience – Part 4 14Mar2009 b) Larger fibers, heavily myelinated, located circumferentially c) Smaller fibers, unmyelinated, located circumferentially d) Larger fibers, heavily unmyelinated, located circumferentially e) Smaller fibers, heavily myelinated, located centrally f) Larger fibers, heavily myelinated, located centrally 6.1) Which of the following anesthetics would be the best choice for a short-term Bier Block procedure of the distal arm? a) Tetracaine b) Bupivacaine c) Lidocaine d) Etidocaine e) Ropivocaine 6.2) Which of the following local anesthetics is NOT a vasodilator and is associated with cerebral hemorrhage, cardiac arrhythmia, and myocardial infarction? a) Cocaine b) Tetracaine c) Lidocaine d) Prilocaine e) Bupivicaine 6.3) Which of the following is NOT true of local anesthetics? a) Can depress cortical inhibitory pathways leading to convulsions, visual and auditory disturbances b) Can pool in the cauda equina and interfere with axonal transport c) Can excite normal pacemaker activity and conduction via chloride channels d) Are required in smaller doses for anesthesia of pregnant patients 7.1) Which of the following anesthetics may cause abnormal conversion of hemoglobin to methemoglobin, leading to hematologic toxicity? a) Cocaine b) Tetracaine c) Lidocaine d) Prilocaine e) Bupivicaine 7.2) Which of the following local anesthetics would NOT be responsible for allergic reactions caused by metabolism to p-aminobenzoic acid derivatives? a) Cocaine b) Lidocaine c) Procaine d) Tetracaine Neuro #7 – Pharmacology: Parkinsonism & Movement Disorders (Chapter 28) 1) Which of the following describes slow, writhing movements? a) Temor b) Chorea c) Athetosis d) Dystonia e) Tics DO NOT DISTRIBUTE - 14 - Neuroscience – Part 4 14Mar2009 2) Which of the following is used to treat Parkinson disorders as it can cross the bloodbrain barrier and is converted to a useful form? a) Dopamine b) Carbidopa c) L-DOPA d) Glutamate e) Glycine 3) What is the role of carbidopa in the drug Sinemet? a) Increase metabolism of levodopa in the periphery b) Increase metabolism of levodopa in the brain c) Decrease metabolism of levodopa in the periphery d) Decrease metabolism of levodopa in the brain 4) Which of the following is NOT a side effect seen in patients receiving L-DOPA treatment? a) Nausea and vomiting b) Bradycardia c) Atrial fibrillation d) Dyskinesias e) Choreoathetosis of the face 5) Fluctuations in clinical state unrelated to timing of L-DOPA doses is known as the “on-off phenomenon.” The recommended clinical method to alleviate this is to have the patient discontinue the drug for 3-21 days (“drug holiday”). a) True b) False, this will not alleviate symptoms c) False, the risks likely outweigh the benefit 6) Which of the following patients should NOT receive L-DOPA? a) Diabetic patients b) Hypertensive patients c) Patients with decreased cardiac output d) Patients with angle-closure glaucoma e) Patients with resting tremors 7) Which of the following is true regarding dopamine agonists in the treatment of Parkinson disease? a) They require enzymatic conversion to an active metabolite b) They have at least two toxic metabolites c) They compete with substances for active transport into the blood d) The have a lower incidence of “on-off phenomenon” fluctuations e) They should be used as a replacement to L-DOPA 8.1) Which of the following is NOT a contraindication of dopamine agonist drugs? a) Psychotic illness b) Dyskinesias c) Recent MI d) Active peptic ulcerations e) Peripheral vascular disease 8.2) Which of the following dopamine agonist drugs is a D3 receptor agonist, used to treat mild and advanced disease Parkinsonism? DO NOT DISTRIBUTE - 15 - Neuroscience – Part 4 14Mar2009 a) Bromocriptine b) Pergolide c) Pramipexole d) Ropinirole 8.3) Which of the following is a monoamine oxidase inhibitor (MAOI) B, which retards the breakdown of dopamine and enhances the effect of L-DOPA? a) Benztropine b) Amantadine c) Selegiline d) Bromocriptine e) Entacopone 8.4) A patient with Parkinson disease is being put on a drug (tolcapone) to help decrease the peripheral metabolism of L-DOPA. They are told they will need to complete a consent form, have liver function tests every two-weeks for a year, and that their urine might turn orange. What type of drug is this? a) Dopamine receptor agonist b) Monoamine oxidase (MAO) inhibitor c) Catechol-O-methyl transferase (COMT) inhibitor d) Anti-viral agent e) Acetylcholine blocker 8.5) A patient with Parkinson disease is given the anti-viral agent amantadine after ensuring the patient does not have a history of seizures or heart failure. What is the mechanism of action for this drug? a) Dopamine receptor agonist b) Selective MAO inhibitor c) Acetylcholine blocker d) Acetylcholine esterase agonist e) Unknown 8.6) Benztropine should be avoided in patients with prostatic hyperplasia, obstructive GI disease, or angle-closure glaucoma. What is the mechanism of action for this drug? a) Dopamine receptor agonist b) Monoamine oxidase (MAO) inhibitor c) Catechol-O-methyl transferase (COMT) inhibitor d) Anti-viral agent e) Acetylcholine blocker 9) In patients with severe Parkinsonism and long-term complications of L-DOPA therapy, such as the on-off phenomenon, a trial of what type of drug may be worthwhile? a) Dopamine receptor agonist b) Monoamine oxidase (MAO) inhibitor c) Catechol-O-methyl transferase (COMT) inhibitor d) Anti-viral agent e) Acetylcholine blocker 10) A patient presents with progressive chorea and dementia. History reveals an autosomal dominant disorder of chromosome 4. Which of the following drugs should be given if the goal is the deplete cerebral dopamine in the nigostriatal pathway by preventing intraneuronal storage? DO NOT DISTRIBUTE - 16 - Neuroscience – Part 4 14Mar2009 a) Reserpine b) Haloperidol c) Perphenazine d) Olanzapine e) Phenothiazine or butyrophenones 11) What is the drug of choice for patients with chronic, multiple tics (Tourette syndrome)? a) Clonidine b) Haloperidol c) Perphenazine d) Penicillamine e) Roprinole 12.1) A 16-year-old patient presents with recurrent hepatic disease. Testing reveals increased urinary copper and a low serum ceruloplasmin (<20mg/dL). Ophthalmologic exam reveals an abnormality, shown here. Along with potassium disulfide, what drug should be given as a chelating agent? a) Clonidine b) Haloperidol c) Reserpine d) Penicillamine e) Roprinole 12.2) A patient presents with an unpleasant creeping discomfort arising deep within the legs that is preventing adequate sleep. Blood test reveals low iron. Which of the following drugs should be used as primary treatment? a) Clonidine b) Haloperidol c) Reserpine d) Penicillamine e) Roprinole Neuro #8 – Pharmacology: Opioids (Chapter 31) 1.1) Most opioids are classified under what schedule per the Controlled Substances Act? a) Schedule I: No accepted medical use or very high risk of abuse b) Schedule II: Has accepted medical use and high risk of abuse c) Schedule III: Has accepted medical use and possible risk of abuse d) Schedule IV: Has accepted medical use and low risk of abuse e) Schedule V: Has accepted medical use and very low risk of abuse Match the opioid receptor subtype with the endogenous opioid peptide affinity: 1.2) Affinity for endorphins a) delta (!) 1.3) Affinity for enkephalins b) kappa (") 1.4) Affinity for dynorphins c) mu (µ) 1.5) Which of the following opioid receptor subtypes functions strongly in modulation of hormones and neurotransmitter release? a) Mu b) Delta c) Kappa DO NOT DISTRIBUTE - 17 - Neuroscience – Part 4 14Mar2009 1.6) Which of the following opioid receptor subtypes has psychomotor effects? a) Mu b) Delta c) Kappa 1.7) Which of the following opioid receptor subtypes is involved in sedation and inhibition of respiration? a) Mu b) Delta c) Kappa 1.8) Which of the following is a strong opioid receptor antagonist? a) Morphine b) Codeine c) Fentanyl (Sublimaze) d) Naloxone (Narcan) e) Nalbuphine (Nubain) 1.9) Although all of the following opiod receptor subtypes are pre-synaptic, which one is also post-synaptic and works by hyperpolarizing second-order pain transmission neurons by increasing K+ conductance and thus inhibiting the post-synaptic action potential? a) Mu b) Delta c) Kappa 2) Pain signals generally follow an fferent nociceptor fiber to the dorsal cord of the spinal cord, decussate and travel to the parabrachial nuclei of the medulla/pons, travel to the ventral caudal thalamus, and finally to the cortex. Which of the following is NOT a site of action for opioids? a) Afferent nociceptor fiber b) Dorsal horn of the spinal cord c) Parabrachial nuclei of the medulla/pons d) Ventral caudal thalamus 3) Which of the following sites of opioid analgesia action on the descending inhibitory pathway is indirectly controlled by the locus caeruleus (site of norepinephrine synthesis)? a) Periaqueductal gray of the midbrain b) Rostral ventral medulla of the medulla/pons c) Dorsal horn of the spinal cord 4) '9-tetrahydrocannavinol (THC) is the main psychoactive ingredient found in marijuana and acts primarily on CB1 cannabinoid receptors. Under certain circumstances, it can interact with the TRPV1 capsaicin receptor, which is normally activated by hot peppers, producing what affect? a) Severe pain b) Unconsciousness c) Analgesia d) Hallucinations e) Intense hunger 5) Which opioid receptor type is most closely associated with opioid-mediated analgesia? a) Mu b) Delta DO NOT DISTRIBUTE - 18 - Neuroscience – Part 4 14Mar2009 c) Kappa 6) A non-tolerant adult goes into respiratory arrest normally at about 60mg of morphine. Opiod tolerant patients may receive dosages of 100mg or 200mg. What dose is required for a maximally tolerant adult to go into respiratory arrest? a) 250mg b) 500mg c) 1g d) 2g e) 3g 7) Which of the following is NOT a sign and symptom of opioid withdrawal? a) Rhinorrhea b) Diarrhea c) Lacrimation d) Yawning e) Chills f) Gooseflesh 8) A patient is given codeine for cough suppression. Which of the following affects has a high degree of tolerance (tolerance can develop)? a) Bradycardia b) Miosis c) Constipation d) Convulsions e) Cough supression 9.1) Opiod antagonists such as naloxone, naltreone, and nalmefene have a high affinity for which of the following receptor sites? a) Mu b) Delta c) Kappa 9.2) A 19-year-old Asian male presents to the Emergency Room responsive only to painful stimuli. General examination reveals pinpoint pupils, injection marks in the antecubital region, and respiratory compromise. Examination of the back reveals several linear red marks and a strong smell of menthol. The patient’s mother says she used Tiger Balm and “coining” to help her son. The clinician gives an opioid antagonist intravenously. The patient awakens quickly and is combative. The clinician has the patient monitored over the next 1-2 hours, as the drug will be metabolized and the patient may again experience respiratory compromise. What drug was administered? a) Flumazenil (Romazicon) b) Fentanyl (Sublimaze) c) Nalmefene (Revix) d) Naltrexone (Revia, Depade) e) Naloxone (Narcan) 9.3) Which of the following is a potent receptor agonist that also blocks NMDA receptors and monoaminergic reuptake, and is used to treat neuropathic pain and opioid abuse? a) Morphine (a phenanthrene) b) Methadone (a phenylheptylamine) c) Meperidine (a phenylpiperidine) DO NOT DISTRIBUTE - 19 - Neuroscience – Part 4 14Mar2009 d) Levorphanol (a morphinan) e) Codeine (a phenanthrene) f) Propoxyphene (a phenylheptylamine) g) Diphenoxylate (a phenylpipere) 9.4) Which of the following is a partial opioid receptor agonist that is usually combined in formulation containing aspirin or acetaminophen? a) Morphine (a phenanthrene) b) Methadone (a phenylheptylamine) c) Meperidine (a phenylpiperidine) d) Levorphanol (a morphinan) e) Codeine (a phenanthrene) f) Propoxyphene (a phenylheptylamine) g) Diphenoxylate (a phenylpipere) 9.5) Which of the following is used with atropine to threat diarrhea? a) Morphine (a phenanthrene) b) Methadone (a phenylheptylamine) c) Meperidine (a phenylpiperidine) d) Levorphanol (a morphinan) e) Codeine (a phenanthrene) f) Propoxyphene (a phenylheptylamine) g) Diphenoxylate (a phenylpipere) 9.6) Which of the following mixed opioid receptor drugs works partially on the mu receptor, has a long duration, and is as effective as methadone? a) Nalbupine (a phenanthrene) b) Buprenorphine (a phenanthrene) c) Butrophanol (a morphinan) d) Pentazocine (a benzomorphan) e) Dezocine (a benzomorphan) 9.7) Nalbupine is a strong agonist of what opioid receptor subtype? a) Mu b) Delta c) Kappa 10.1) Anti-tussives (cough suppressants) should NOT be taken with which of the following, as it could result in hyperpyrexic (high fever) coma? a) Dopamine receptor agonists b) Monoamine oxidase (MAO) inhibitors c) Catechol-O-methyl transferase (COMT) inhibitors d) Anti-viral agents e) Acetylcholine blockers f) Selective serotonin reuptake (SSR) inhibitors g) Anti-fungal agents 10.2) Which of the following anti-tussives works at lower doses than those required for analgesia? a) Codeine b) Dextromethorphan c) Levopropoxyphene DO NOT DISTRIBUTE - 20 - Neuroscience – Part 4 14Mar2009 Neuro #9 – Pharmacology: Drugs Of Abuse (Chapter 32) 1) Which of the following is associated with dependence and not addiction? a) Compulsive drug use despite negative consequences b) Relapsing drug use c) Drug cravings in response to contextual clues d) Withdrawal symptoms 2.1) The mesolimbic pathway connects the ventral tegmental area (VTA) to the nucleus accumbens. The VTA, which is part of the pleasure-reward system, contains neurons of all of the following EXCEPT: a) Dopamine b) GABA c) Glutamate d) Norepinephrine 2.2) Which of the following is NOT a target for dopamine in the mesolimbic and mesocortical pathways? a) Hippocampus b) Prefrontal cortex c) Amygdala d) Locus coeruleus e) Nucleus accumbens 2.3) Where is GABA (gamma-aminobutyric acid) produced? a) Hippocampus b) Prefrontal cortex c) Amygdala d) Locus coeruleus e) Nucleus accumbens f) Ventral tegmental area Match the drug of abuse with the main molecular target (receptor): 3.1) µ-O Receptor (Gio) – Agonist a) Opiates 3.2) CB1 Receptor (Gio) – Agonist b) Nicotine 3.3) 5-HT2A Receptor (Gq) – Partial agonist c) Benzodiazepines 3.4) nACh Receptor (#2$2) – Agonist d) Cocaine 3.5) GABA-A Receptor – Agonist e) Ecstasy (MDMA) 3.6) DAT, SERT, NET – Inhibitor f) LSD, mescaline 3.7) SERT > DAT, NET – Reverse transport g) Cannabinoids 3.8) Which of the following, along with amphetamines, has the highest relative risk (RR) of addiction (score of 5 meaning highly addictive, with 1 meaning non-addictive)? a) Opioids b) Nicotine c) LSD d) Cocaine e) Ecstasy 3.9) Which of the following, along with PCP and ketamine, is considered RR=1, or nonaddictive? a) Cannabinoids DO NOT DISTRIBUTE - 21 - Neuroscience – Part 4 14Mar2009 b) Nicotine c) LSD d) Cocaine e) Ecstasy 4.1) Dronabinol, an appetite stimulant, and nabilone, an anti-emetic, are analogs of which of the following types of drugs? a) Cannabinoids b) Nicotine c) LSD d) Cocaine e) Ecstasy 4.2) Which of the following involves buproprion (Wellbutrin) as a treatment option and is associated with Alzheimer dementia with loss of acetylcholine-releasing neurons from the nucleus basalis of Meynert? a) Amphetamines b) Benzodiazepines c) Cannabinoids d) Cocaine e) Nicotine 4.3) Which of the following inhibits voltage gated Na+ channels in the PNS, blocks uptake of dopamine in the CNS, and can lead to hyperthermic coma with overdose? a) Opioids b) Benzodiazepines c) Cannabinoids d) Cocaine e) Nicotine 4.4) A 17-year-old female presents to the Emergency Room after passing out at a “rave” party. She is hot to the touch and very diaphoretic. Examination finds two bottles of water, glow sticks, and an infant pacifier (to prevent teeth grinding). Which of the following is the most likely? a) Cannabinoids b) Amphetamine c) LSD d) Cocaine e) Ecstasy Neuro #10 – Neurology: Disorders Of Consciousness (Chapter 1) 1) Which of the following states of consciousness is usually irreversible? a) Coma b) Delirium c) Acute confusional state d) Dementia 2) Which of the following is NOT a clinical feature of delirium? a) Sleepy, disoriented, inattentive b) Non-responsive to verbal or painful stimuli c) Agitation with drowsiness DO NOT DISTRIBUTE - 22 - Neuroscience – Part 4 14Mar2009 d) Autonomic changes (sweating, pallor) e) Hallucinations and motor abnormalities (tremor) 3) Which of the following is true of dementia, but not acute confusional state (delirium)? a) Acute or sub-acute time course b) Usually irreversible c) Autonomic hyperactivity usually presents d) Level of consciousness not impaired 4) Which of the following is NOT usually a contributor to delirium and dementia? a) Cardiovascular disease b) Diabetes c) Seizure d) Head trauma e) Hyperthyroidism 5.1) Which of the following changes in vital signs is seen with multi-infarct dementia, and not hypothyroidism dementia? a) Hypothermia b) Hypertension c) Hypotension d) Bradycardia 5.2) A 26-year-old female presents with neurologic symptoms. History reveals an extremely active sex life. Examination reveals small pupils bilaterally that accommodate to a near object but do not react to light (light-near dissociation). Treatment with penicillin is started. Which of the following is the most likely? a) Brain tumor b) Neuro syphilis c) Hepatic degeneration d) Creutzfeldt-Jakob disease (CJD) e) Huntington disease 5.3) A patient presents with a new onset headache and neurologic symptoms. Suspecting an increase in intracranial pressure, the clinician performs an ophthalmologic exam using a slit lamp and finds papilledema. Which of the following is most likely? a) Brain tumor b) Neuro syphilis c) Hepatic degeneration d) Creutzfeldt-Jakob disease (CJD) e) Wilson disease 5.4) A 50-year-old physician presents with depression. Physical exam reveals a “wingbeating” tremor of the wrists when they are dorsiflexed (extended). The patient responds “yes” to all of the CAGE questions. Which of the following is the most likely cause of this patient’s tremor? a) Brain tumor b) Neuro syphilis c) Hepatic degeneration d) Creutzfeldt-Jakob disease (CJD) e) Huntington disease f) Depression DO NOT DISTRIBUTE - 23 - Neuroscience – Part 4 14Mar2009 6) Which of the following involves patient resistance to passive extension of the knee with the hip flexed? a) Nuchal rigidity (meningismus) b) Brudzinski sign c) Kernig sign d) Thomas test e) Straight leg raise test f) Rovsing sign 7) Kayser-Fleisher rings around the iris are associated with which of the following? a) Brain tumor b) Neuro syphilis c) Hepatic degeneration d) Creutzfeldt-Jakob disease (CJD) e) Wilson disease 8.1) A positive Myerson sign is often an early sign of Parkinson disease. Which of the following describes the primitive (frontal release sign) reflex for this? a) Palmar: elicited by stroking the skin of the patients palm, if the reflex is present, the patients finger will close around the examiners and often times with the inability to let go b) Plantar: flexion and adduction of the toes in response to stimulation of the foot sole c) Palmomental: scratching along the length of the palm of the hand results in contraction of the ipsilateral chin (mentalis) and perioral (orbicularis) muscles d) Suck: sucking movements upon gentle tapping of the lips e) Snout: tapping on the lips results in their protrusion f) Glabellar: elicited by repeated tapping of the forehead causing persistent blinking 8.2) Which of the following is a disorder of articulation that spares oral and written language comprehension and written expression? a) Aphasia b) Dysarthria c) Agraphesthesia d) Astereognosia e) Apraxia f) Pseudobulbar palsy 9.1) Which step of the Mini Mental Status Examination (MMSE) involves re-drawing the image shown here? a) Orientation b) Registration c) Attention & Calculation d) Recall e) Language 9.2) Which step of the Mini Mental Status Examination (MMSE) involves using serial 7’s or spelling the word “world” backwards? a) Orientation b) Registration DO NOT DISTRIBUTE - 24 - Neuroscience – Part 4 14Mar2009 c) Attention & Calculation d) Recall e) Language 10) Visual hallucinations are commonly seen in ____ and auditory hallucinations are commonly seen in ____. a) Acute confusional states; Psychiatric disorders b) Psychiatric disorders; Acute confusional states c) Acute confusional states; Psychiatric disorders and acute confusional states d) Psychiatric disorders; Acute confusional states and psychiatric disorders 11.1) A patient presents with the ability to comprehend language and speak fluently. However, they are unable to repeat phrases fluently. Which of the following best describes their aphasia? a) Receptive (Wernicke) b) Conductive (Arcuate fasciculus) c) Expressive (Broca) d) Global (Broca & Wernicke) 11.2) A patient presents with neurological problems after a recent stroke. They are asked to raise their hand, but respond verbally with “the body is a unit.” When asked what they did today, they respond “repetition is the key to mastery.” Which of the following best describes their aphasia? a) Receptive (Wernicke) b) Conductive (Arcuate fasciculus) c) Expressive (Broca) d) Global (Broca & Wernicke) 12) Orientation to place and time is a test for which form of memory? a) Remote memory b) Recent memory c) Immediate memory 13) Which of the following is defined as weakness affecting the lower extremities? a) Hemiparesis b) Hemiplegia c) Paraparesis d) Paraplegia e) Quadriparesis f) Quadriplegia 14) Chronic alcoholics are at an increased risk of life-threatening hypoglycemia and bacterial meningitis. During ethanol withdrawal, these patients are at risk for Wernicke Encephalitis, and thus should be given: a) Diazepam, low dose b) Diazepam, high dose c) Chlorodiazepoxide d) Thiamine e) Disulfiram 15) Which of the following is an abnormal finding in spinal fluid analysis? a) Pressure 70-200mmH2O b) Glucose >45mg/dL DO NOT DISTRIBUTE - 25 - Neuroscience – Part 4 14Mar2009 c) Protein >45mg/dL d) Glutamine <25mcg/dL e) Appearance clear 16.1) A lumbar puncture reveals slightly cloudy CSF, normal pressure, an increase in mononuclear cells, a decrease in glucose, an increase in protein, and a positive India Ink stain on smear. Which of the following is the most likely? a) Bacterial meningitis b) Tuberculosis (TB) meningitis c) Fungal meningitis d) Viral meningitis e) Leptomeningeal carcinomatosis f) Subarachnoid hemorrhage 16.2) A lumbar puncture reveals mononuclear cell infiltration with a drastic decrease in glucose. Culture is negative but cytology is positive for an abnormality. History reveals painful ejaculations over the past few months. Results of a PSA test are pending. Which of the following is the most likely? a) Bacterial meningitis b) Tuberculosis (TB) meningitis c) Fungal meningitis d) Viral meningitis e) Leptomeningeal carcinomatosis f) Subarachnoid hemorrhage 16.3) A patient presents with the “worst headache of my life.” A spinal tap reveals pink CSF with increased pressure and moderately increased protein. Smear and culture are negative. Which of the following is the most likely? a) Bacterial meningitis b) Tuberculosis (TB) meningitis c) Fungal meningitis d) Viral meningitis e) Leptomeningeal carcinomatosis f) Subarachnoid hemorrhage 17) A teenager presents with confusion, nausea, hypotension, and papillary constriction. A Foley catheter is put in place to relieve urinary retention. Examination finds “track marks” on the anterior left arm. To rule-out a pontine hemorrhage, which of the following should be given to the patient? a) Furosemide (Lasix) b) Mannitol c) Dobutamine d) Flumazenil e) Naloxone 18) A runner presents to the medical tent at the finish line of a marathon (26.2 miles). The patient is delirious, confused, and combative. The patient says he drank water at ever rest station along the way but did not have any electrolyte rehydration drinks. An IV is started and the patient is given hypertonic saline. The infusion should not be done rapidly to prevent which of the following? a) Retinal detachment DO NOT DISTRIBUTE - 26 - Neuroscience – Part 4 14Mar2009 b) Subarachnoid hemorrhage c) Central pontine myelinolysis d) Absence seizures e) Atrial fibrillation 19) A patient presents with several neurologic symptoms including delirium and hallucinations. ECG reveals a prolonged QT interval (ventricular repolarization). Percussion of the facial nerve anterior to the ear produces hyper-excitability (Chvostek/Weiss sign). While obtaining a blood pressure, hyper-reflexia of the wrist occurs (Trousseau sign). Which of the following is the most likely? a) Hypokalemia b) Hyperkalemia c) Hypocalcemia d) Hypercalcemia e) Hyponatremia 20.1) A patient presents with an electric-shock sensation down their spine with neck flexion (Lhermitte sign). Lab tests show thrombocytopenia with giant neutrophils and pernicious anemia with a positive Shilling test. IM injections of cyanocobalamin are started. Which of the following does this patient have? a) Reye syndrome b) Hepatic encephalopathy c) Wenicke encephalopathy d) B12 deficiency e) Korsakoff amnestic syndrome 20.2) A heavy-drinker presents with cognitive disturbances and asterixis. Examination reveals brisk ocular reflexes. Blood testing reveals increased ammonia; lactulose is given to decrease pH. Which of the following is the most likely? a) Reye syndrome b) Hepatic encephalopathy c) Wenicke encephalopathy d) B12 deficiency e) Korsakoff amnestic syndrome 20.3) Which of the following is associated with a triad of ophthalmoplegia, ataxia, and confusional state? a) Reye syndrome b) Hepatic encephalopathy c) Wenicke encephalopathy d) B12 deficiency e) Korsakoff amnestic syndrome 20.4) Which of the following is associated with varicella, influenza B, and the use of aspirin to treat febrile illness in children? a) Reye syndrome b) Hepatic encephalopathy c) Wenicke encephalopathy d) B12 deficiency e) Korsakoff amnestic syndrome DO NOT DISTRIBUTE - 27 - Neuroscience – Part 4 14Mar2009 20.5) A heavy-drinker presents with memory loss, confusion, and confabulation. Which of the following is the most likely? a) Reye syndrome b) Hepatic encephalopathy c) Wenicke encephalopathy d) B12 deficiency e) Korsakoff amnestic syndrome 21.1) Which of the following forms of bacterial meningitis is associated with a petechial rash? a) S. pneumonia b) Haemophilis c) N. meningitidis d) Listeria e) M. tuberculosis f) T. pallidum 21.2) Isoniazid is a first-line drug used for which of the following? a) Bacterial meningitis b) Neuro syphilis c) Lyme disease d) Tuberculosis e) AIDS 21.3) Which of the following is used to treat Lyme disease (B. burgdorferi)? a) Acyclovir b) Doxycycline c) Penicillin d) Isoniazid e) Amphotericin B 21.4) Which of the following forms of meningitis seen in AIDS patients affects the basal ganglia and is declining in frequency with the use of trimethoprim-sulfamethoxole for P. carinii? a) Cryptococcus b) HSV/VZV c) CMV d) Toxoplasmosis e) Primary CNS lymphoma 22) Which of the following is often associated with a lateral skull fracture (e.g. fracture at the pterion) without loss of consciousness? a) Concussion b) Epidural hematoma c) Subdural hematoma d) Intracerebral hematoma 23) Which of the following reflexes is most commonly lost with increasing age? a) Biceps b) Brachioradialis c) Triceps d) Patella DO NOT DISTRIBUTE - 28 - Neuroscience – Part 4 14Mar2009 e) Achilles 24) Which of the following is an irreversible cause of dementia? a) Normal pressure hydrocephalus b) Intracranial mass lesion c) Vitamin B12 deficiency d) Hypothyroidism e) Creutzfeldt-Jakob disease Match the disease with the description: 25.1) Severe depression with cognitive changes a) Alzheimer disease 25.2) 50-70 years old after minor head trauma b) Creutzfeldt-Jakob disease (CJD) 25.3) Pseudobulbar palsy, dysarthria, dysphagia c) Normal pressure hydrocephalus 25.4) Dementia, gait ataxia, incontinence d) Vascular dementia 25.5) From corneal transplant or growth hormones e) Chronic subdural hematoma 25.6) German heritage, neurofibtrillary tangles, beta amyloid f) Pseudodementia Neuro #11 – Neurology: Headache & Facial Pain (Chapter 2) 26) Which of the following is NOT a pain sensitive structure of the head? a) Venous sinuses b) Dura mater c) Arteries of the Circle of Willis d) Eyes, ears, nose, mouth e) Teeth 27.1) Which of the following is NOT true regarding headaches? a) Migraines can be exacerbated with oral contraceptives b) Sharp, lancing pain is associated with trigeminal neuralgia c) Ice-pick-like pain is associated with cluster headaches d) The majority of migraine attacks are bilateral cluster headaches e) Most patients with tension headaches report bilateral pain f) Pain in the V1 area that is burning in quality is associated with postherpetic neuralgia (Herpes zoster) 27.2) Which of the following is typically seen with cluster headaches? a) Recent weight loss b) Dyspnea c) Visual disturbances d) Nausea and vomiting e) Rhinorrhea and lacrimation 28) A patient presents with a “thunderclap headache.” Lumbar puncture reveals CSF that is blood-tinged and centrifuge substrate is tinged yellow. A subarachnoid hemorrhage is suspected. Which of the following is the most likely cause? a) Polycystic kidney disease b) Coarctation of the aorta c) Arteriovenous malformation (AVM) d) Ruptured “berry” aneurysm e) Thrombosis of the middle meningeal artery 29.1) Which of the following will abort an acute attack of trigeminal neuralgia, also known as tic douloureux or suicide disease? DO NOT DISTRIBUTE - 29 - Neuroscience – Part 4 14Mar2009 a) Carbamazepine b) Oxcarbazepine c) Phenytoin d) Lamotrigine e) Baclofen 29.2) A 55-year-old patient presents with pain over the left forehead. Examination reveals a decreased blink reflex on the left. History reveals a recent infection. Which of the following is suspected if the clinician begins treatment with acyclovir? a) Pseudotumor cerebri b) Trigeminal neuralgia c) Postherpetic neuralgia d) Giant cell arteritis e) Intracranial mass 29.3) High dose prednisone will show very fast relief of symptoms, such as headaches and jaw claudication, when given for which of the following? a) Pseudotumor cerebri b) Trigeminal neuralgia c) Postherpetic neuralgia d) Giant cell arteritis e) Intracranial mass 29.4) In pseudotumor cerebri, the intracranial pressure is usually self-limiting over several months, especially with treatment including acetazolamine and furosemide. Which of the following is characteristic of the epidemiology of this disease? a) 16-year-old pregnant drinker b) 24-year-old extremely athletic male c) 26-year-old obese female d) 35-year-old diabetic male e) 75-year-old swimmer 29.5) A patient wakes up with a headache that reaches maximal intensity within a few seconds and is accompanied by nausea and vomiting. Which of the following is the most likely? a) Pseudotumor cerebri b) Trigeminal neuralgia c) Postherpetic neuralgia d) Giant cell arteritis e) Intracranial mass 30) Which of the following is distinguished from a stroke by gradual onset and spontaneous resolution? a) Tension headache b) Migraine c) Cluster headache 31.1) A patient presents with bilateral throbbing pain, cervical muscle contraction, and scalp tenderness. History reveals similar headaches after eating chocolate or take-out Chinese food. Which of the following is the most likely? a) Cluster headache b) Tension headache DO NOT DISTRIBUTE - 30 - Neuroscience – Part 4 14Mar2009 c) Migraine with aura (classical) d) Migraine without aura (common) 31.2) All of the following drugs are used prophylatically for migraines. Which of the following is a calcium channel blocker? a) Amitriptyline b) Propranolol c) Methergine d) Verapamil e) Phenytoin 32.1) A 25-year-old man presents with brief, very severe, unilateral, and constant nonthrobbing headaches. They occur most often when the patient is drinking alcohol with friends. An MRI shows activation of the ipsilateral hypothalamic gray. Which of the following is the most likely? a) Cluster headache b) Tension headache c) Migraine with aura (classical) d) Migraine without aura (common) 32.2) A 20-year-old patient presents with complaints of frequent non-throbbing bilateral occipital head pain. The patient denies any nausea, vomiting, or prodromal visual disturbance. The patient is given amitriptyline for prophylaxis and told to take aspirin or acetaminophen for acute attacks. Which of the following is the most likely? a) Cluster headache b) Tension headache c) Migraine with aura (classical) d) Migraine without aura (common) Neuro #12 – Neurology: Disorders Of Equilibrium (Chapter 3) 33) Which of the following describes the expected result of caloric testing with cold water in the right ear? a) Nystagmus with slow phase toward the right then fast beating away (left) b) Nystagmus with slow phase toward the left then fast beating toward (right) c) Nystagmus with fast beating toward the right then slow phase away (left) d) Nystagmus with fast beating toward the left then slow phase toward (right) 34.1) A 35-year-old male patient presents with repeated episodes of vertigo lasting from minutes to days. History reveals the vertigo is accompanied by tinnitus and that his hearing is getting worse. Hydrochlorothiazide (HCTZ) is prescribed. Which of the following is the likely cause? a) Benign positional vertigo b) Ménière disease c) Vestibular neuronitis d) Cerebellopontine angle tumor 34.2) A patient presents with a cerebellopontine angle tumor. Examination reveals café au lait spots. Which of the following chromosomes is related to this autosomal dominant disorder? a) 3 b) 5 DO NOT DISTRIBUTE - 31 - Neuroscience – Part 4 14Mar2009 c) 11 d) 17 f) 21 34.3) A patient presents with complains of nausea and lightheadedness when sleeping on one side. The clinician performs a series of maneuvers while holding the patient’s head in an attempt to help move debris from the endolymph. Which of the following is causing the patient’s symptoms? a) Benign positional vertigo b) Ménière disease c) Vestibular neuronitis d) Cerebellopontine angle tumor 34.4) A patient presents with spontaneous attacks of vertigo accompanied by nausea and vomiting. In the primary care clinic, the patient has another episode and lays on her right side, refusing to move. Caloric testing is defective in the right ear. Nystagmus is present toward the left side. The patient is prescribed a trial of prednisone for which of the following? a) Benign positional vertigo b) Ménière disease c) Vestibular neuronitis d) Cerebellopontine angle tumor Neuro #13 – Neurology: Disturbances Of Vision (Chapter 4) 35.1) A patient is asked to look left but the left eye does not move. Which of the following cranial nerves is damaged? a) Left CN III b) Right CN III c) Left CN IV d) Right CN IV e) Left CN VI f) Right CN VI 35.2) Each lateral gaze center in the paramedian pontine reticular formation (PPRF) mediates ____ conjugate horizontal gaze via connections to the ____ abducens nerve nucleus and the ____ oculomotor nerve nucleus. a) Contralateral; Contralateral; Contralateral b) Contralateral; Contralateral; Ipsilateral c) Ipsilateral; Ipsilateral; Ipsilateral d) Ipsilateral; Ipsilateral; Contralateral 36) Which of the following is specifically associated with oculomotor palsy? a) Diplopia b) Ophthalmoplegia c) Ptosis d) Hemianopsia e) Quadrantanopsia 37.1) Damage to the Meyer Loop in the temporal lobe causes which of the following? a) Central scotoma b) Ipsilateral blindness DO NOT DISTRIBUTE - 32 - Neuroscience – Part 4 14Mar2009 c) Bitemporal hemianopia d) Contralateral hemianopia e) Contralateral homonymous superior quadrantanopia f) Contralateral homonymous inferior quadrantanopia g) Contralateral homonymous hemianopia 37.2) Posterior cerebral artery occlusion damaging the optic radiation causes: a) Central scotoma b) Ipsilateral blindness c) Bitemporal hemianopia d) Contralateral hemianopia e) Contralateral homonymous superior quadrantanopia f) Contralateral homonymous inferior quadrantanopia g) Contralateral homonymous hemianopia 37.3) A pituitary tumor would cause: a) Central scotoma b) Ipsilateral blindness c) Bitemporal hemianopia d) Contralateral hemianopia e) Contralateral homonymous superior quadrantanopia f) Contralateral homonymous inferior quadrantanopia g) Contralateral homonymous hemianopia 38) Which of the following would be seen bilaterally during an ophthalmologic exam with increased intracranial pressure and/or with brain tumors? a) Arteriovenous nicking b) Flame hemorrhages c) Hard exudates d) Papilledema e) Cotton wool spots 39) The pupillary light reflex involves accommodation and reaction to light and is associated with what cranial nerve(s)? a) II b) III c) II, III d) III, IV, VI e) II, III, IV, VI 40.1) During a swinging-flashlight test, the pupils constricts less when the light is swung from the left eye to the right eye. There is no anisocoria. The right eye still senses light but the reaction is diminished. Which of the following describes this patient? a) Non-reactive pupils b) Light-near dissociation c) Argyll-Robertson pupils d) Tonic pupils e) Horner syndrome f) Afferent pupillary defect (Marcus Gunn pupil) 40.2) Examination of a patient with suspected Holmes-Adie syndrome reveals that the left pupil is larger than the right and the left pupil reacts sluggishly to changes in DO NOT DISTRIBUTE - 33 - Neuroscience – Part 4 14Mar2009 illumination or accommodation. Anisocoria becomes less marked during the duration of the exam. Which of the following describes this presentation? a) Non-reactive pupils b) Light-near dissociation c) Argyll-Robertson pupils d) Tonic pupils e) Horner syndrome f) Afferent pupillary defect (Marcus Gunn pupil) 40.3) Local diseases of the iris, oculomotor nerve compression, and optic nerve disorders are associated with which of the following clinical presentations? a) Non-reactive pupils b) Light-near dissociation c) Argyll-Robertson pupils d) Tonic pupils e) Horner syndrome f) Afferent pupillary defect (Marcus Gunn pupil) 40.4) A patient presents with dry skin on the left side of the face, minor ptosis, and a small pupil on the left. Which of the following is the most likely? a) Non-reactive pupils b) Light-near dissociation c) Argyll-Robertson pupils d) Tonic pupils e) Horner syndrome f) Afferent pupillary defect (Marcus Gunn pupil) 40.5) Neurosyphilis, diabetes, optic nerve disorders, and tumors compressing the midbrain tectum are associated with which of the following clinical presentations? a) Non-reactive pupils b) Light-near dissociation c) Argyll-Robertson pupils d) Tonic pupils e) Horner syndrome f) Afferent pupillary defect (Marcus Gunn pupil) 40.6) A patient presents with light-near dissociation. Pupils are small, irregular in shape, and unequal in size. A lesion is suspected in the region of the Edinger-Westphal nucleus. Which of the following describes the clinical presentation? a) Non-reactive pupils b) Marcus Gunn pupil c) Argyll-Robertson pupils d) Tonic pupils e) Horner syndrome 41.1) A patient presents with sudden, painless, monocular vision loss and pre-monitory ocular symptoms. Treatment options are unsuccessful. Which of the following is the most likely? a) Amaurosis fugax b) Optic neuritis c) Anterior ischemic optic neuropathy DO NOT DISTRIBUTE - 34 - Neuroscience – Part 4 14Mar2009 41.2) A patient presents with transient monocular blindness (amaurosis fugax) due to emboli in the carotid artery bifurcation. Which of the following should be used to treat this patient? a) Prednisone b) Methylprednisolone c) Amitriptyline d) Phenytoin e) Aspirin 41.3) A demyelinating disease leads to optic neuritis. Which of the following should be given to the patient initially? a) Prednisone b) Methylprednisolone c) Amitriptyline d) Phenytoin e) Aspirin 42) Which of the following is characteristic of diabetic third nerve palsy, and not aneurismal third nerve palsy? a) Involvement of the posterior communicating artery b) Offending location is in the subarachnoid space c) Pupillary sparing d) Lack of CN VI involvement e) Lack of CN IV involvement Neuro #14 – Neurology: Motor Deficits (Chapter 5) 43) Where are the cell bodies of upper motor neurons (UMN) located? a) Cerebral cortex b) Brain stem c) Spinal cord d) A & B e) B & C 44) During muscle strength testing, a patient receives a great deal of 3/5 scores. Which of the following is NOT a peripheral neuropathy that could be leading to this weakness? a) Guillian-Barre syndrome b) Shellfish poisoning c) Diphtheria d) Botulism e) Arsenic poisoning 45) A patient presents with complaints of progressive muscle weakness of the proximal arms/legs and fatigue. History reveals an organ transplantation surgery three years ago. Which of the following drugs that cause myopathy is the patient most likely taking? a) Beta-blockers b) Zidovudine c) Penicillamine d) Corticosteroids e) Chloroquine 46.1) Which of the following is associated with the clasp-knife reflex? DO NOT DISTRIBUTE - 35 - Neuroscience – Part 4 14Mar2009 a) Hypertonia b) Hypotonia c) Areflexia d) Hyporeflexia e) Hyperreflexia 46.2) Which of the following is associated with the Hoffmann sign, where eliciting finger flexion causes thumb flexion? a) Hypertonia b) Hypotonia c) Areflexia d) Hyporeflexia e) Hyperreflexia 46.3) If the abdominal reflex (T8-12) is elicited by stroking the left upper quadrant of the abdomen, which direction will the umbilicus tend to move with abdominal muscle contraction? a) Toward the right upper quadrant (RUQ) b) Toward the left upper quadrant (LUQ) c) Toward the right lower quadrant (RLQ) d) Toward the left lower quadrant (LLQ) 46.4) The cremaster reflex is associated with what root levels? a) T8-10 b) L1-2 c) L2-4 d) S1-S2 e) L4-S1 47) Which of the following signs is indicative of a lower motor neuron lesion? a) Positive Babinski sign b) Spasticity and hyperreflexia c) Weakness or paralysis d) Loss of superficial abdominal reflexes e) Wasting and fasciculations 48) A patient presents with an extensive, but unilateral cord lesion (hemisection). In Brown-Séquard syndrome, motor deficit is accompanied by ____ impairment of vibration and position sense as well as ____ loss of pain and temperature appreciation. a) Ipsilateral; Ipsilateral b) Ipsilateral; Contralateral c) Contralateral; Contralateral d) Contralateral; Ipsilateral 49) Which of the following is NOT true of myopathic disorders? a) Weakness is usually most marked distally b) No muscle wasting is present c) Deep tendon reflexes are intact d) No sensory loss e) No sphincter disturbances 50.1) When checking serum enzymes for muscle disease, which of the following will show the greatest increase and is the most useful in following the course of the disease? DO NOT DISTRIBUTE - 36 - Neuroscience – Part 4 14Mar2009 a) Creatine kinase b) Aldolase c) Lactic acid dehydrogenase d) Transaminases 50.2) A muscle biopsy specimen shows atrophied fibers in groups, with adjacent groups of larger, uninvolved fibers. Which of the following describes the underlying weakness? a) Neurogenic b) Myopathic c) A & B d) None of the above 51.1) Which of the following is associated with the development of focal scattered areas of demyelination followed by a reactive gliosis? a) Huntington disease b) Parkinson disease c) Multiple sclerosis d) Down syndrome e) Amyotrophic lateral sclerosis 51.2) A 24-year-old female from Wisconsin presents with relapsing and recurring muscle weakness. Studies have revealed two lesions involving different regions of the central white matter at different times. Which of the following types of drugs can help reduce the relapse rate for this patient? a) Corticosteroids b) Mitozantrone c) Cyclophosphamide d) Interferon " e) Amantadine 52.1) A patient presents with fever, backache, and tenderness along a spinal nerve root distribution. MRI with gadolinium enhancement shows an epidural abscess. Which of the following is the most likely cause? a) Staphylococcus aureus b) Niesseria gonorrhoeae c) Bacillus anthracis d) Borrelia burgdorferi e) Moraxella catarrhalis 52.2) A patient presents with spastic paraparesis with brisk tendon reflexes. History reveals they recently had flaccidity. Examination reveals pain and temperature appreciation are lost. Abdominal examination reveals a pulsating mass and ultrasound reveals a dissecting aortic aneurysm. Which of the following arteries is involved in this patient’s neurologic symptoms? a) Posterior inferior cerebellar artery b) Anterior inferior cerebellar artery c) Posterior spinal artery d) Anterior spinal artery e) Basilar artery 52.3) A patient presents with pain that radiates down the neck when they cough. CSF is yellow (xanthochromic) with increased protein, increased WBC count, and decreased DO NOT DISTRIBUTE - 37 - Neuroscience – Part 4 14Mar2009 glucose. CT is used instead of a Queckenstedt maneuver and shows a lesions. Which of the following is the most likely? a) Cervical disk prolapse b) Poliomyelitis c) Multiple sclerosis d) Tetanus e) Tumor with cord compression 52.4) A patient presents with prodromal phase fever, myalgia, and weakness that is asymmetric in distribution after traveling to a third-world country. History reveals they were not immunized due to their parent’s fear of diseases caused by immunization. A viral infection is suspected. Which of the following is the most likely? a) Influenza b) Poliomyelitis c) Multiple sclerosis d) Tetanus e) Herpes zoster 52.5) In patients with lumbar disk prolapsed, cyclobenzaprine (Flexoril) is given for: a) Pain b) Nerve regeneration c) Muscle spasm d) Nausea e) Incontinence 52.6) A 24-year-old male presents after falling from a tree and trying to grasp a branch. Their grasping arm is flexed at the elbow and fingers are gripping (Klumpke palsy). Which of the following nerve roots are damaged? a) C3-C4 b) C5-C6 c) C6-C8 d) C8-T1 e) T2-T4 52.7) A 24-year-old male presents after a motorcycle accident. They were wearing a helmet but landed on their shoulder and side of their head, causing drastic side-bending of the cervical spine. They present in a “waiter’s tip” position with arm extended, internally rotated, and wrist flexed (Erb-Duchenne palsy). Which of the following nerve roots are damaged? a) C3-C4 b) C5-C6 c) C6-C8 d) C8-T1 e) T2-T4 53.1) Which of the following progresses slowly and leads to kyphoscoliosis and contractures? a) Infantile spinal muscular atrophy (Werdig-Hoffman Disease) b) Intermediate spinal muscular atrophy (Chronic Werdig-Hoffman Disease) c) Juvenile spinal muscular atrophy (Kugelberg-Welander Disease) 53.2) Which of the following affects mostly proximal limb muscles? DO NOT DISTRIBUTE - 38 - Neuroscience – Part 4 14Mar2009 a) Infantile spinal muscular atrophy (Werdig-Hoffman Disease) b) Intermediate spinal muscular atrophy (Chronic Werdig-Hoffman Disease) c) Juvenile spinal muscular atrophy (Kugelberg-Welander Disease) Match the following diseases with UMN, LMN, or mixed: 53.3) Progressive bulbar palsy a) Upper motor neuron disease 53.4) Pseudobulbar palsy b) Lower motor neuron disease 53.5) Progressive spinal muscular atrophy c) Mixed UMN/LMN disease 53.6) Primary lateral sclerosis 53.7) Amyotrophic lateral sclerosis (ALS) 53.8) Which of the following drugs can reduce mortality and slow the progression of amyotrophic lateral sclerosis (ALS)? a) Anticholinergic b) Isoniazid c) Phenytoin d) Carbamazepine e) Riluzole 54.1) A child presents with anemia, constipation, abdominal pain, gum discoloration, and severe sensory absence in the arms and legs. They are started on IV calcium disodium. Which of the following is the most likely cause of this patient’s symptoms? a) Arsenic poisoning b) Gold poisoning c) Lead poisoning d) Incomplete Bell palsy e) Complete Bell palsy 54.2) Which of the following is associated with Ramsay-Hunt syndrome? a) Arsenic poisoning b) Gold poisoning c) Lead poisoning d) Bell palsy e) Horner syndrome Match the description with the disease: 55.1) Autosomal dominant, chromosome 19, 70% mortality a) Myasthenia gravis 55.2) Proximal limb-girdle muscles, Raynaud phenomenon, elevated CK b) Botulism 55.3) CTG repeat on chromosome 19 c) Duchenne muscular dystrophy (DMD) 55.4) X-linked, pseudohypertrophy of the calves d) Malignant hyperthermia 55.5) Diplopia, ptosis, profound weakness, home-canning e) Myotonic dystrophy 55.6) Pernicious anemia, give guanidine hydrochloride f) Poliomyositis 55.7) Muscle nicotinic ACh-R antibodies, Thymoma g) Myasthenic syndrome Neuro #15 – Neurology: Disorders Of Somatic Sensation (Chapter 6) 56.1) Which of the following is described as stocking-and-glove sensory loss? a) Parathesia b) Dysesthesia c) Anesthesia d) Hypesthesia e) Hyperesthesia DO NOT DISTRIBUTE - 39 - Neuroscience – Part 4 14Mar2009 f) Mononeuropathy g) Polyneuropathy 56.2) Which of the following denotes an unpleasant sensation produced by a stimulus that is usually painless? a) Parathesia b) Dysesthesia c) Anesthesia d) Hypesthesia e) Hyperesthesia f) Mononeuropathy g) Polyneuropathy 57) When a patient says a part of their body is numb, they most likely mean that part of their body is experiencing: a) Heaviness b) Weakness c) Deadness d) Any of the above 58.1) Which of the following tracts corresponds to “1” in the spinal cord section here? a) Lateral spinothalamic b) Anterior spinothalamic c) Corticospinal d) Rubrospinal e) Fasciculus gracilis 58.2) What tract corresponds to “5”? a) Lateral spinothalamic b) Anterior spinothalamic c) Corticospinal d) Rubrospinal e) Fasciculus gracilis 59.1) Which of the following dermatomes is incorrect? a) Male nipple: T4 b) Umbilicus: T10 c) Inguinal region: L1 d) Lateral calf: L4 e) Lateral foot: S1 59.2) What root level most corresponds to the dermatome of the middle finger? a) C5 b) C6 c) C7 d) C8 e) T1 59.3) What nerve sensory distribution accounts for the lateral (thumb) side of the back (dorsum) of the hand? a) Ulnar nerve b) Median nerve c) Radial nerve DO NOT DISTRIBUTE - 40 - Neuroscience – Part 4 14Mar2009 d) Median antebrachii cutaneous nerve e) Musculocutaneous nerve 59.4) What nerve sensory distribution accounts for the space between the big and second toe on the sole of the foot? a) Peroneal b) Sural c) Saphenous d) Lateral femoral cutaneous 59.5) What nerve sensory distribution accounts for medial malleolus and medial calf? a) Peroneal b) Sural c) Saphenous d) Lateral femoral cutaneous 60) Which of the following would NOT be seen with an occlusion of the anterior spinal artery? a) Bilateral spastic paresis below the lesion b) Bilateral loss of pain and temperature below the lesion c) Bilateral Horner syndrome d) Bilateral spastic paralysis of innervated muscles 61) Which of the following describes involvement of several nerves with a random, discontinuous (non-overlapping) pattern? a) Mononeuropathy simplex b) Mononeuropathy multiplex c) Polyneuropathy 62) A ____ is usually traumatic or ischemic in origin. Work-related carpal tunnel syndrome is an example of a ____. a) Polyneuropathy; Polyneuropathy b) Polyneuropathy; Mononeuropathy c) Mononeuropathy; Polyneuropathy d) Mononeuropathy; Mononeuropathy 63) Guillain-Barré syndrome is characterized by acute or subacute polyneuropathy, usually following an infection of ____. Treatment includes plasmapharesis and immunoglobulin therapy. a) ETEC (E. coli) b) C. jejuni c) Rotovirus d) H. pylori e) EHEC (E. coli) 64) Which of the following describes the most common type of diabetes mellitus neuropathy? a) Mononeuropathy of the arm(s) b) Mononeuropathy of the leg(s) c) Polyneuropathy of the arms d) Polyneuropathy of the legs 65.1) A patient presents with severe, prolonged hypothyroidism with myxedema. Which of the following nerves is the most likely to have mononeuropathy do to the myxedema? DO NOT DISTRIBUTE - 41 - Neuroscience – Part 4 14Mar2009 a) Ulnar b) Median c) Radial d) Peroneal e) Femoral 65.2) Which of the following nerves is most likely to be damaged with compression in the axilla? a) Ulnar b) Median c) Radial d) Peroneal e) Femoral 65.3) Thoracic outlet syndrome caused by a cervical rib usually affects the C5-C6 distribution. a) True b) False, it affects the C8-T1 distribution 65.4) A patient presents with a dragging left foot (foot drop). History reveals traumatic injury near the lateral knee. Which of the following nerves was damaged? a) Lateral femoral cutaneous b) Femoral c) Peroneal d) Radial e) Median 66) A college student presents with new onset of distal parasthesia and weakness in the extremities. Examination reveals increased reflexes and Barber Chair Phenomenon (Lhermitte sign). History reveals a recent change to a vegetarian diet. If subacute combined degeneration is suspected, what vitamin will be deficient? a) Vitamin A b) Vitamin B3 c) Vitamin B6 d) Vitamin B12 e) Vitamin E 67) Which of the following would result in causalgia, not reflex sympathetic dystrophy? a) Soft tissue trauma b) Bone fracture c) Nerve trauma d) Myocardial infarction e) Stroke 68) Which of the following maneuvers would most close the intervertebral foramen of the cervical spine on one side in an attempt to recreate radiculopathy (Spurling test)? a) Rotation and flexion b) Rotation and side-bending c) Rotation and extension d) Side-bending and flexion 69) Which of the following is/are the best modalities of treatment for low back and neck pain as it relates to trauma (musculoskeletal pain)? DO NOT DISTRIBUTE - 42 - Neuroscience – Part 4 14Mar2009 a) Codeine and physical therapy b) Diazepam, paracetamol, and ice c) Spinal decompression therapy d) Surgical treatment e) Heat, bed rest, and NSAIDs 70.1) A patient presents with burning and shooting pain along a dermatome. Herpes zoster is diagnosed and the patient is given cortisone for the rash and analgesics for pain. After treatment, which of the following is the most common complication that this patient may have (treated with Phenytoin)? a) Ramsay-Hunt syndrome b) Motor neuropathy c) Meningitis d) Encephalitis e) Postherpetic neuralgia 70.2) A 24-year-old man with HIV presents to the ED with intensifying left ear pain, swelling, and dizziness. His hearing in his left ear is muffled compared to that in his right ear. There is no history of trauma or allergic reactions, and the patient does not take any medications. The physical examination is notable for an obviously erythematous and edematous left ear (see image), as well as a subtle left facial droop and weakness. What is the diagnosis? a) Ramsay-Hunt Syndrome b) Bell Palsy c) Trigeminal neuralgia d) Otitis externa 70.3) Which of the following is NOT a clinical finding in Ramsay-Hunt syndrome? a) Ear pain b) A vesicular rash within the ear canal and/or pinna c) Facial weakness d) Nystagmus e) Involvement of the cornea 70.4) Treatment of Ramsay-Hunt syndrome may involve all of the following EXCEPT: a) Lubricating ear drops b) Consultation with an infectious disease specialist c) Prednisone d) Amoxicillin e) Acyclovir Neuro #16 – Neurology: Movement Disorders (Chapter 7) 71) Which of the following is associated with the superior cerebellar peduncle? a) Postural tremor b) Resting tremor c) Intention tremor 72) Damage to the contralateral subthalamic nucleus causes: a) Hemiballismus b) Asterixis DO NOT DISTRIBUTE - 43 - Neuroscience – Part 4 14Mar2009 c) Chorea d) Tics e) Dystonia Match the disease with the description: 73.1) Streptococcus infection, arteritis a) Familial and essential tremor 73.2) More troublesome at night, use dopaminergic drugs b) Huntington disease 73.3) CAG repeats, dementia, chorea c) Sydenham chorea 73.4) Striato-thalamo-cortico pathways, clonidine for tics d) Spasmodic torticollis 73.5) Affects hands and voice, but not usually legs e) Gilles de la Tourette syndrome 73.6) Tendency for neck to twist to one side f) Restless leg syndrome 74) A patient presents with hypokinesia. Lab tests reveal the presence of Lewy bodies. Which of the following is the most likely? a) Huntington disease b) Parkinson disease c) Multiple sclerosis d) Down syndrome e) Amyotrophic lateral sclerosis 75.1) A patient presents with Parkinsonism symptoms. Which of the following would NOT be seen if the syndrome were drug-induced? a) Bradykinesia b) Resting tremor c) Rigidity d) Postural instability 75.2) Neuroleptic Malignant Syndrome, which mimics malignant hyperthermia, is a rare complication of which of the following drugs? a) Reserpine b) Haloperidol c) Perphenazine d) Olanzapine e) Metoclopramide 75.3) Tardive Dyskinesia is seen often in institutionalized individuals with long-term use of which of the following drugs? a) Reserpine b) Haloperidol c) Perphenazine d) Olanzapine e) Metoclopramide Neuro #17 – Neurology: Seizures & Syncope (Chapter 8) 76) Which of the following is true of seizures and not syncope? a) Occur in upright or sitting positions b) Associated with physical exertion c) Prompt return of consciousness with full lucidity d) Preceded by aura or focal sensory phenomena e) Produces flaccid unresponsiveness 77) Which of the following describes recurrent episodes of abnormal neuronal discharge? DO NOT DISTRIBUTE - 44 - Neuroscience – Part 4 14Mar2009 a) Syncope b) Absence seizure c) Atonic seizure d) Tonic-clonic seizure e) Epilepsy 78) Which of the following would NOT likely cause seizures? a) Head trauma b) Meningitis c) Drug overdose d) Hypoglycemia e) Hyperkalemia 79) A 2-year-old child is brought into the Emergency Room because it was “shaking and turned blue.” Physical exam reveals a high fever. What drug should be given for prolonged convulsions and to possibly reduce recurrence? a) Phenytoin b) Zaleplon c) Phenobarbital d) Diazepam e) Flumazenil 80.1) Which of the following involves loss of consciousness without loss of postural tone and may have subtle motor manifestations like eye blinking or lip smacking? a) Tonic-clonic seizure (grand mal) b) Absence seizure (petit mal) c) Myoclonic seizure d) Simple partial seizure e) Complex partial seizure 80.2) Which of the following involves the temporal lobe and may have affective (fear) sensations, cognitive (déjà vu) sensations, sensory (olfactory) hallucinations, and involuntary motor activities (automatisms)? a) Tonic-clonic seizure (grand mal) b) Absence seizure (petit mal) c) Myoclonic seizure d) Simple partial seizure e) Complex partial seizure 80.3) Myoclonic, or shock-like contracting, seizures may be associated with all of the following EXCEPT: a) Todd paralysis b) Unverricht-Lundborg disease c) Lafora body disease d) Neuronal ceroid lipofuscinosis e) Mitochondrial encephalomyopathy f) Sialidosis 80.4) Which of the following is associated with autonomic symptoms (pallor, flushing, sweating, vomiting, borborygmi, incontinence) with preserved consciousness? a) Tonic-clonic seizure (grand mal) b) Absence seizure (petit mal) DO NOT DISTRIBUTE - 45 - Neuroscience – Part 4 14Mar2009 c) Myoclonic seizure d) Simple partial seizure e) Complex partial seizure 80.5) Which of the following mimics opisthotonos and may involves a crying or moaning sound and tongue trauma? a) Tonic-clonic seizure (grand mal) b) Absence seizure (petit mal) c) Myoclonic seizure d) Simple partial seizure e) Complex partial seizure 81) Status epilepticus is defined as a seizure lasting longer than ____ minutes or that recur so frequently that consciousness is not regained. They are considered a medical emergency as permanent brain damage can result from hyperpyrexia, circulatory collapse, or excitoxic neuronal damage. a) 3 b) 5 c) 10 d) 15 e) 30 f) 60 82) Primary generalized seizures affect ____ cerebral hemisphere(s) and secondary generalized seizures affect ____ cerebral hemisphere(s). a) One; One b) One; Both c) Both; One d) Both; Both 83) Which of the following is true of absence seizures, when compared with complex partial seizures? a) Preceded by aura b) Lasts seconds c) Has automatisms 84) Which of the following should be done if a patient has new onset seizures after the age of 25 or if neurological exam is abnormal? a) EEG (abnormal spikes, polyspikes, spike-wave complexes) b) Serium calcium c) Fasting glucose d) Serum FTA-ABS e) Brain MRI 85) Which of the following is NOT true regarding therapeutic management of epilepsy? a) Epilepsy drugs may cause seizures in normal (non-epileptic) patients b) Establish a diagnosis of epilepsy before starting therapy c) Choose the right drug for the seizure type d) Treat seizures rather than the serum drug level e) Evaluate two drugs at a time to help prevent symptoms Match the type of seizure with the drugs that can be used for treatment: 86.1) Generalized seizures a) Valproic acid, Ethosuximide DO NOT DISTRIBUTE - 46 - Neuroscience – Part 4 14Mar2009 86.2) Partial seizures b) Phenytoin, Carbamazepine 86.3) Myoclonic seizures c) Valproic acid, Clonazepam 86.4) Absence seizures d) Phenytoin, Phenobarbitol, Carbamazepine 87) Along with vigabatrin, which of the following should be avoided when treated generalized seizures? a) Gabapentin b) Lamotrigine c) Leviracetam d) Topiramate e) Zonisamide 88) For status epilepticus of unknown origin, what should be given? a) Lorazepam b) Diazepam c) Phenytoin d) Phenobarbital e) Dextrose 89) Which of the following have been linked to increased neural tube defects if given to a pregnant patient? a) Folic acid b) Phenytoin and ethosuximide c) Valproic acid and carbamazepine d) Clonazepam and diazepam e) Propofol and midazolam 90) Which of the following forms of pseudoseizure involves the patient recognizing that the spells are self-induced, but not the motivation for them? a) Conversion/somatization disorder b) Factitious disorder c) Malingering disorder 91.1) Which of the following forms of syncope is associated with blood donation or seeing the sight of blood? a) Vasovagal syncope b) Neurocardiogenic syncope c) Carotid sinus syncope d) Micturition syncope e) Subclavian steal syncope 91.2) A 22-year-old man presents with a fainting spell during a medical school interview. History reveals the patient does not normally wear a suit and thus wore an old suit that was now very tight fitting. To rule out superventricular tachycardia, the clinician performs a massage procedure. Which of the following is the clinician testing for? a) Vasovagal syncope b) Neurocardiogenic syncope c) Carotid sinus syncope d) Micturition syncope e) Subclavian steal syncope f) None of the above DO NOT DISTRIBUTE - 47 - Neuroscience – Part 4 14Mar2009 Neuro #18 – Neurology: Stroke (Chapter 9) 92) What time period differentiates strokes from transient ischemic attacks (TIAs)? a) 3-hours b) 6-hours c) 12-hours d) 24-hours e) 48-hours 93) A stroke with deficits that reach maximum at onset is common in what subtype? a) Embolic stroke b) Arterial thrombotic stroke c) Subarachnoid hemorrhage 94) Which of the following is common in posterior circulation ischemia as oppose to anterior circulation ischemia? a) Aphasia b) Apraxia c) Vomitting d) Hemiparesis e) Visual field defects 95) Which of the following is a common site for thrombotic strokes, not for embolic strokes? a) Middle cerebral artery b) Apex of basilary artery c) Posterior cerebral artery d) Internal carotid artery 96) Which of the following involves toxic effects of blood on brain tissue, leading to cerebral dysfunction? a) Embolic stroke b) Arterial thrombotic stroke c) Subarachnoid hemorrhage 97) A patient presents with a suspected stroke. They have asterognosia, Wernicke aphasia, and contralateral homonymous hemianopia. Which of the following is the most likely location of the stroke? a) Anterior cerebral artery b) Right superior division of middle cerebral artery c) Left superior division of middle cerebral artery d) Right interior division of middle cerebral artery e) Left interior division of middle cerebral artery Match the clinical signs seen with the location of the stroke: 98.1) Upper limb hemiparesis, Broca aphasia a) Internal carotid artery 98.2) Oculomotor palsy, hemiplegia, decorticate position b) Basilar artery (thrombus) 98.3) Locked-in Syndrome, impaired horizontal gaze c) Basilar artery (emboli) 98.4) Monocular blindness, ipsilateral retinal ischemia d) Middle cerebral artery 99) Which of the following arteries is the most susceptible to a lacunar stroke, often caused by microatheroma or lipohyalinosis? a) Anterior cerebral artery b) Middle cerebral artery DO NOT DISTRIBUTE - 48 - Neuroscience – Part 4 14Mar2009 c) Internal carotid artery d) Basilar artery e) Lenticulostriate artery 100) Which of the following is NOT a reason why CT scans are more advantageous than MRI scans? a) They are widely available b) They are fast c) They can distinguish between ischemia and hemorrhage d) They are cheaper e) They are superior in demonstrating early ischemic infarcts 101) Which of the following modalities is used to detect stenosis or occlusion of the internal carotid artery non-invasively? a) Cerebral angiogram b) Echocardiogram c) Electrocardiogram d) Ultrasonography 102) Which of the following treatment modalities for transient ischemic attacks (TIAs) has the best benefit-to-risk ratio? a) Antiplatelet therapy (Aspirin) b) Anticoagulants (Heparin, Warfarin) c) Carotid endarterectomy d) Angioplasty with intraluminal stents e) Extracranial/Intracranial bypass 103) A patient is rushed to the hospital after an apparent stroke. Paramedics state the patient was abnormal for all three sections of the Cincinnati Stroke Scale. A CT scan shows a probably acute ischemic stroke and blood pressure is not hypertensive. From the onset of symptoms, what is time window for giving tissue plasminogen activator (tPA)? a) 30-minutes b) 1-hour c) 3-hours d) 6-hours e) 24-hours 104) What is the major cause of intracerebral hemorrhage? a) Cocaine use b) Amphetamine use c) Chronic hypertension d) Berry aneurysms e) Trauma to the lateral skull Neuro #19 – Neurology: Coma (Chapter 10) 105) Which of the following is NOT done immediately at the onset of a coma? a) Electrocardiogram (ECG) b) Arterial blood gas (ABG) c) Serum glucose, electrolytes d) Manage airway, breathing, circulation e) Start an IV line to administer drugs DO NOT DISTRIBUTE - 49 - Neuroscience – Part 4 14Mar2009 106) Which of the following is NOT given immediately to a coma patient if the coma is of unknown origin? a) Dextrose b) Thiamine c) Naloxone d) Flumazenil Match the clinical signs with the possible cause of the coma: 107.1) Coma preceeded by confusional state a) Subarachnoid hemorrhage 107.2) Symptoms days or weeks prior to coma b) Tumor, abscess 107.3) Rapid progression from hemispheric signs to coma c) Intracerebral hemorrhage 107.4) Sudden onset coma d) Metabolic derangement 108) A 24-year-old racecar driver presents to the Emergency Room with head trauma after a car crash. A basilar skull fracture is suspected. Which of the following would NOT likely be seen? a) Periorbital ecchymoses (Raccoon eyes) b) Perimastoid ecchymoses (Battle sign) c) Hemotympanum (blood in ears) d) CSF otorrhea or rhinorrhea e) Decorticate or decerebrate positioning Match the clinical presentation with the level of CNS involvement: 109.1) Fixed midsized pupils, no motor response a) Early diencephalon 109.2) Decerebrate, fixed midsized pupils b) Late diencephalon 109.3) Decorticate with painful stimuli, small pupils c) Midbrain 109.4) Semi-purposeful response to painful stimuli, small pupils d) Pons/Medulla 110) Which of the following would be seen with a lesion of the thalamus (decorticate), not with a lesion of the midbrain (decerebrate)? a) Patient unconscious b) Shoulders internally rotated c) Leg extension d) Elbows flexed 111) A patient presents with ipsilateral pupillary dilation, impaired adduction of the eye, and eventual loss of consciousness. Which of the following herniations is most likely? a) Central herniation b) Uncal herniation c) Cerebellar herniation d) Tosillar herniation e) Cingulate herniation 112) Which of the following breathing patterns are indicative of a pontomedullary lesion? a) Cheyne-Stokes b) Central hyperventilation c) Ataxic d) Gasping e) A & B f) C & D DO NOT DISTRIBUTE - 50 - Neuroscience – Part 4 14Mar2009 113) A patient presents with cerebral edema. The clinician starts an IV line running hypertonic saline (3%). Which of the following is an osmotic diuretic agent that can help relieve intracranial pressure? a) Dexamethasone b) Dextrose c) Mannitol d) Dobutamine e) Milrinone 114.1) A patient is being treated for a cardiac arrest in the Emergency Room. Shortly after treatment, the patient experiences opisthotonic posturing with seizures and fecal incontinence. Examination reveals rapidly dilating pupils. Which of the following is the mostly likely? a) Subdural hematoma b) Epidural hematoma c) Intracerebral hemorrhage d) Brain abscess e) Pontine hemorrhage f) Global cerebral ischemia 114.2) Which of the following should be given for an insulin overdose? a) Dexamethasone b) Dextrose c) Mannitol d) Dobutamine e) Milrinone 114.3) A patient presents with apopletic onset coma and hyperthermia. Examination revealed ocular bobbing prior to unconsciousness. Which of the following is the most likely? a) Subdural hematoma b) Epidural hematoma c) Intracerebral hemorrhage d) Brain abscess e) Pontine hemorrhage f) Global cerebral ischemia 114.4) Patients with which of the following are the most likely to have intracerebral hemorrhages? a) Diabetes b) Epilepsy c) Hypertension d) Bradycardia e) Pneumonia 114.5) Which of the following is seen in patients with cyanotic congenital heart disease or recent head trauma? a) Global cerebral ischemia b) Epidural hematoma c) Intracerebral hemorrhage d) Brain abscess DO NOT DISTRIBUTE - 51 - Neuroscience – Part 4 14Mar2009 e) Pontine hemorrhage 115) A 16-year-old patient presents with a coma. Oculocephalic testing reveals an absence of extraocular movements with preservation of pupillary light reflex. Pupils are normal in size. The patient’s mother thinks the patient took the pills she uses for acute management of her status eilepticus. Which of the following would be most beneficial for this patient? a) Alcohol detoxification b) IV naloxone c) IV flumazenil d) IV dextrose e) Oral activated charcoal 116) Which of the following would NOT be seen in a patient with hepatic encephalopathy? a) Hyporeflexia b) Asterixis c) Somnolence or delerium d) Respiratory alkalosis e) Hyperventilation A patient presents in a coma of known origin. Brainstem reflexes are absent and the patient is unresponsive. For diagnosis of brain death, match the description with the length of time for confirmation: 117.1) Anoxic brain injury without confirmatory EEG a) 6-hours 117.2) Brain injury with confirmatory isoelectric EEG b) 12-hours 117.3) Brain injury without confirmatory EEG c) 24-hours Neuro #20 – Neurology: Neurological Investigations (Chapter 11) 118.1) Which of the following is NOT an indication for lumbar puncture? a) Diagnosis of meningitis b) Diagnosis of intracranial mass lesion c) Administration of radiologic contrast media d) Administration of intrathecal medication e) To reduce CSF pressure 118.2) The frequency of post-lumbar-puncture headaches is related to: a) The use of NSAIDs b) The use of caffeine c) The size of the spinal needle d) The site of lumbar puncture e) Lying in the recumbent position 119) The use of electroencephalography (EEG) is the least useful in which of the following? a) Evaluating epilepsy b) Assessing prognosis of seizures c) Managing tonic-clonic status epilepticus d) Detecting structural brain lesions e) Diagnosis of neurologic disorders DO NOT DISTRIBUTE - 52 - Neuroscience – Part 4 14Mar2009 120) Electromyography (EMG) and a nerve conduction study (NCS) are often used in conjunction to help understand and diagnose various neurologic problems. a) True b) False, EMG is no longer used c) False, NCS is no longer used d) False, neither of these tests are used 121) Diagnosis of which of the following requires an MRI, not just a CT scan? a) Dementia b) Tumor c) Stroke d) Subarachnoid hemorrhage e) Multiple sclerosis 122) Cerebral angiography is NOT useful in which of the following cases? a) Arteriovenous malformations b) Aneurysms > 3mm in diameter c) Visualizing the circle of Willis d) Detecting subdural hematomas e) Screening for aortic stenosis Neuro #21 – Amyotrophic Lateral Sclerosis (Case 3) 1) A 45-year-old male presents with marked progressive weakness in his hands and arms. Which of the following is the most likely? a) Poliomyelitis b) Multiple sclerosis c) Myasthenia gravis d) Amyotrophic lateral sclerosis e) Guillain-Barré syndrome 2) Which of the following diagnostic laboratory tests and studies is most abnormal in a patient with amyotrophic lateral sclerosis? a) Blood chemistry b) Cerebrospinal fluid c) CT scan of brain d) CT scan of cervical spine e) Electromyogram 3) Which of the following is an inflammatory demyelinating disease, usually involving the peripheral nerves, with high incidence in young adults? a) Poliomyelitis b) Multiple sclerosis c) Myasthenia gravis d) Amyotrophic lateral sclerosis e) Guillain-Barré syndrome 4) Which of the following describes the lesion seen in amyotrophic lateral sclerosis? a) Bilateral loss of touch, vibration, and tactil sense from lower limbs due to lesion of the fasciculus gracilis b) Bilateral loss of pain and temperature one level below due to lesion of the ventral white commissure (spinothalamic tract) DO NOT DISTRIBUTE - 53 - Neuroscience – Part 4 14Mar2009 c) Combined UMN and LMN lesion of corticospinal tract d) Bilateral Horner syndrome, loss of bladder control due to lesion of corticospinal tract e) Ipsilateral flaccid paralysis, ipsilateral spastic paresis, contralateral loss of pain and touch due to loss of spinothalamic tract Neuro #22 – Alzheimer Disease (Case 35) 1) An elderly patient presents with progressive memory loss, confusion, agitation, and forgetfulness. Which of the following is the most likely? a) Brain tumor b) Chronic alcoholism c) Hypothyroidism d) Neurosyphilis e) Alzheimer disease 2) Which of the following diagnostic laboratory tests and studies would be the least useful for an elderly patient with progressive memory loss? a) Thyroid function test b) VDRL test c) Thiamine assay d) Electrocardiogram e) Vitamin B12 assay 3) Which of the following is associated with cold intolerance, weight gain, mental slowness, thinning hair, and edema of the face/hands? a) Brain tumor b) Chronic alcoholism c) Hypothyroidism d) Neurosyphilis e) Alzheimer disease Match the disease with the deficient vitamin: 4.1) Night blindness a) Vitamin B1 (thiamin) 4.2) Megaloblastic anemia b) Vitamin B3 (niacin) 4.3) Pellegra c) Vitamin B12, folic acid 4.4) BeriBeri d) Vitamin A 4.5) Which of the following cranial nerves is NOT normally affected in Alzheimer disease? a) CN I b) CN II c) CN II, IV, VI d) CN V e) CN VII f) CN VIII 4.6) Which of the following causes of dementia is autosomal recessive and involves the accumulation of galactocerebroside in the brain? a) Alzheimer disease b) Huntington disease c) Parkinson disease DO NOT DISTRIBUTE - 54 - Neuroscience – Part 4 14Mar2009 d) Krabbe disease e) Metachromatic leukodystrophy 5) Which of the following is an autosomal recessive lysosomal storage disease that results in demylination and progressive dementia, caused by an accumulation of sulfatide in the brain, kidneys, liver, and peripheral nerves? a) Alzheimer disease b) Huntington disease c) Parkinson disease d) Krabbe disease e) Metachromatic leukodystrophy Neuro #23 – Down Syndrome (Case 43) 1) A 4-week-old male patient presents for a well baby examination. He is the product of an uncomplicated pregnancy of a 40-year-old mother. The mother reports having a low #-fetoprotein (AFP) level during pregnancy. On physical examination, the patient is noted to have a flat occiput; white spots in his iris (Brushfield spots); a large, protruding tongue; small, low-set ears; broad, short feet and hands; a flexion crease across his palm (simian crease); clindodactyly of the fifth digit; a systolic ejection mumur; split S2; and hypertonia. Which of the following is most likely? a) Cri du chat syndrome (5p-) b) Edwards syndrome (trisomy 18) c) Fragile X syndrome d) Down syndrome (trisomy 21) e) Patau syndrome (trisomy 13) f) Fetal alcohol syndrome 2) Which of the following would NOT be seen in diagnostic laboratory tests and studies of a patient with Down syndrome? a) Trisomy 21 b) Endocardial cushing defect c) Maternal serum AFP decreased d) Maternal unconjugated estriol decreased e) Maternal human chorionic gonadotropin (hCG) decreased 3.1) Which of the following is associated with low-set ears, rocker-bottom feet, and micrognathia? a) Cri du chat syndrome (5p-) b) Edwards syndrome (trisomy 18) c) Fragile X syndrome d) Down syndrome (trisomy 21) e) Patau syndrome (trisomy 13) 3.2) Which of the following is associated with large ears, macrocephaly, a long face, and macroorchidism? a) Cri du chat syndrome (5p-) b) Edwards syndrome (trisomy 18) c) Fragile X syndrome d) Down syndrome (trisomy 21) e) Patau syndrome (trisomy 13) DO NOT DISTRIBUTE - 55 - Neuroscience – Part 4 14Mar2009 4) Which of the following is a genetic imprinting disease (methylation inactivation) involving paternal 15q deletion? a) Angelman syndrome b) Prader-Willi syndrome c) Down syndrome d) Cri du chat syndrome e) Patau syndrome James Lamberg DO NOT DISTRIBUTE - 56 - Neuroscience – Part 4 AnswerKey Neuro #1 1) B 2) C 3.1) D 3.2) C 3.3) A 3.4) E 3.5) B 3.6) A 3.7) C 4) D 5) E 6) C 7.1) C 7.2) B 8.1) D 8.2) B 8.3) A 8.4) A 8.5) D 8.6) F 8.7) E 8.8) C Neuro #2 1) A 2) D 3.1) C 3.2) A 4.1) A 4.2) D 5) B 6.1) C 6.2) B 7.1) B 7.2) C 8) E 9.1) D 9.2) C 10) E 11) D 12) D 13.1) E 13.2) D 13.3) E 4.1) A 4.2) C 4.3) C 4.4) C 4.5) A 5) E 6.1) B 6.2) C 6.3) A 7) C 8) A 9.1) D 9.2) C 9.3) E 9.4) E 9.5) E 9.6) A 10) C Neuro #6 1) E 2.1) D 2.2) C 3) D 4) D 5) A 6.1) C 6.2) A 6.3) C 7.1) D 7.2) B Neuro #7 1) C 2) C 3) C 4) B 5) C 6) C 7) D 8.1) B 8.2) C 8.3) C 8.4) C 8.5) E 8.6) E 14Mar2009 9) C 10) A 11) B 12.1) D 12.2) E Neuro #8 1.1) B 1.2) C 1.3) A 1.4) B 1.5) B 1.6) C 1.7) A 1.8) D 1.9) A 2) C 3) C 4) C 5) A 6) D 7) B 8) E 9.1) A 9.2) E 9.3) B 9.4) E 9.5) G 9.6) B 9.7) C 10.1) B 10.2) A Neuro #9 1) D 2.1) D 2.2) D 2.3) E 3.1) A 3.2) G 3.3) F 3.4) B 3.5) C 3.6) D 3.7) E 3.8) D Neuro #3 1.1) C 1.2) E 1.3) D 2.1) C 2.2) E 3) D 4) E 5.1) B 5.2) A 5.3) C 6.1) E 6.2) A Neuro #4 1.1) B 1.2) D 1.3) A 1.4) B 1.5) C 1.6) C 1.7) B 1.8) A 1.9) A 1.10) B 1.11) C 1.12) D 1.13) D 2.1) E 2.2) A 2.3) E 2.4) E 2.5) C 2.6) C 3) B Neuro #5 1.1) A 1.2) E 1.3) C 1.4) D 1.5) A 1.6) B 2) C 3) A DO NOT DISTRIBUTE - 57 - Neuroscience – Part 4 3.9) C 4.1) A 4.2) E 4.3) D 4.4) E Neuro #10 1) D 2) B 3) D 4) E 5.1) B 5.2) B 5.3) A 5.4) C 6) C 7) E 8.1) F 8.2) B 9.1) E 9.2) C 10) A 11.1) B 11.2) A 12) B 13) C 14) D 15) C 16.1) C 16.2) E 16.3) F 17) E 18) C 19) C 20.1) D 20.2) B 20.3) C 20.4) A 20.5) E 21.1) C 21.2) D 21.3) B 21.4) D 22) B 23) E 24) E 25.1) F 25.2) E 25.3) D 25.4) C 25.5) B 25.6) A Neuro #11 26) B 27.1) D 27.2) E 28) D 29.1) C 29.2) C 29.3) D 29.4) C 29.5) E 30) B 31.1) D 31.2) D 32.1) A 32.2) B Neuro #12 33) A 34.1) B 34.2) D 34.3) A 34.4) C Neuro #13 35.1) E 35.2) D 36) C 37.1) E 37.2) G 37.3) C 38) D 39) B 40.1) F 40.2) D 40.3) A 40.4) E 40.5) B 40.6) C 41.1) C 41.2) E 41.3) B 42) C Neuro #14 43) A 44) D 45) D 46.1) B 46.2) E 46.3) B 46.4) B 47) E 48) B 49) A 50.1) A 50.2) A 51.1) C 51.2) D 52.1) A 52.2) D 52.3) E 52.4) B 52.5) C 52.6) D 52.7) B 53.1) B 53.2) C 53.3) B 53.4) A 53.5) B 53.6) A 53.7) C 53.8) E 54.1) C 54.2) D 55.1) D 55.2) F 55.3) E 55.4) C 55.5) B 55.6) G 55.7) A Neuro #15 56.1) G 14Mar2009 56.2) B 57) D 58.1) E 58.2) A 59.1) D 59.2) C 59.3) C 59.4) A 59.5) C 60) D 61) B 62) D 63) B 64) D 65.1) B 65.2) C 65.3) B 65.4) D 66) D 67) C 68) C 69) E 70.1) E 70.2) A 70.3) E 70.4) D Neuro #16 71) C 72) A 73.1) C 73.2) F 73.3) B 73.4) E 73.5) A 73.6) D 74) B 75.1) B 75.2) B 75.3) E Neuro #17 76) D 77) E 78) E 79) D DO NOT DISTRIBUTE - 58 - Neuroscience – Part 4 80.1) B 80.2) E 80.3) A 80.4) D 80.5) A 81) E 82) C 83) B 84) E 85) E 86.1) D 86.2) B 86.3) C 86.4) A 87) A 88) E 89) C 90) B 91.1) A 91.2) C Neuro #18 92) D 93) A 94) C 95) D 96) C 97) E 98.1) D 98.2) C 98.3) B 98.4) A 99) E 100) E 101) D 102) A 103) C 104) C Neuro #19 105) A 106) D 107.1) D 107.2) B 107.3) C 107.4) A 108) E 109.1) D 109.2) C 109.3) B 109.4) A 110) D 111) B 112) F 113) C 114.1) F 114.2) B 114.3) E 114.4) C 114.5) D 115) C 116) A 117.1) C 117.2) A 117.3) B Neuro #20 118.1) B 118.2) C 119) C 120) A 121) E 122) D Neuro #21 1) D 2) E 3) E 4) C Neuro #22 1) E 2) D 3) C 4.1) D 4.2) C 4.3) B 4.4) A 4.5) D 4.6) D 5) E Neuro #23 1) D 2) E 3.1) B 3.2) C 4) B 14Mar2009 DO NOT DISTRIBUTE - 59 -
Copyright © 2024 DOKUMEN.SITE Inc.